You are on page 1of 64

Chemical Equilibrium 1

PHYSICAL CHEMISTRY

Target : JEE MAIN AND ADVANCED 2022

Exercise Sheet

CHEMICAL EQUILIBRIUM

Index
Theory 02 - 34
Exercise 35 - 60
Answer Key 61 - 64

www.etoosindia.com
Plot No. 46, In front of Skyline Apartments, Corner Building,
Rajeev Gandhi Nagar, Kota, (Rajasthan) Pin code : 324005.
Help Desk : 09214233303 | 09214233343 |
2 Chemistry for JEE

CHEMICAL EQUILIBRIUM
1. INTRODUCTION
Whenever we hear the word Equilibrium immediately a picture arises in our mind an object under the
influence of two opposing forces. For chemical reactions also this is true.Areaction also can exist in a
state of equilibrium balancing forward and backward reactions.
Symbolic representation of any chemical change in terms of reactants and products is called chemical
reaction.
Types of chemical reaction :
(a) On the basis of physical state
Homogeneous reactions Heterogeneous reaction
All reactants and products are in same phase Reactants and products are in two or more phase
N2(g) + 3H2(g) 2NH3(g) Zn(s) + CO2(g) ZnO(s) + CO(g)

(b) On the basis of direction


Reversible reaction Irreversible reaction
(i) Chemical reaction in which products Chemical reaction in which products cannot
can be converted back into reactants be convert back into reactants
H2 + I2 2HI Zn + H2SO4  ZnSO4 + H2
(ii) Proceed in forward as well as backward Proceed only in forward direction
direction
(iii) These attain equilibrium These do not attain equilibrium
(iv) Reactant are never completely Reactants are nearly completely converted
converted into products into products
(v) Generally thermal dissociations are held in Generally thermal decompositions are held
closed vessel in open vessel
PCl5(g) PCl3(g) + Cl2(g) 2KClO3(s)  2KCl(s) + 3O2(g)

(c) On the basis of speed


Fast reactions Slow reactions
(i) Generally these reactions are ionic Generally these reactions are molecular
in nature in nature
HCl + NaOH  NaCl + H2O H2 + I2  2HI
Acid Base Salt Water

(d) On the basis of heat


Exothermic reaction Endothermic reaction
(i) Heat is evolved in these type of chemical Heat is absorbed in these type of chemical
reactions reactions
R  P + x kcal R  P – x kcal
Chemical Equilibrium 3

It is an experimental fact that most of the process including chemical reactions,


when carried out in a closed vessel, do not go to completion. Under these
conditions, a process starts by itself or by initiation, continues for some time
at diminishing rate and ultimately appears to stop. The reactants may still be
present but they do not appear to change into products any more.

2. EQUILIBRIUM AND ITS DYNAMIC NATURE


2.1 Definition : "Equilibrium is the state at which the concentration of reactants and products do not change
with time. i.e. concentrations of reactants and products become constant."
2.2 Characteristics : Following are the important characteristics of equilibrium state,
Products

Concentration

Reactants

Time

(i) Equilibrium state can be recognised by the constancy of certain measurable properties such as
pressure, density, colour, concentration etc. by changing these conditions of the system, we can
control the extent to which a reaction proceeds.
(ii) Equilibrium state can only be achieved in close vessel, but if the process is carried out in an open
vessel equilibrium state cannot be attained because in an open vessel, the reverse process may
not take place.
(iii) Equilibrium state is reversible in nature.
(iv) Equilibrium state is also dynamic in nature. Dynamic means moving and at a microscopic level,
the system is in motion. The dynamic state of equilibrium can be compared to water tank having
an inlet and outlet. Water in tank can remain at the same level if the rate of flow of water from
inlet (compared to rate of forward reaction) is made equal to the rate of flow of water from
outlet (compared to rate of backward reaction). Thus, the water level in the tank remains constant,
though both the inlet and outlet of water are working all the time.
Fo
rw
ar
dr
Rate of reaction

ea
ctio
n

(v) At equilibrium state, Equilibrium


n
ctio

State
rea
rd
wa
ck
Ba

Rate of forward reaction = Rate of backward reaction


Time
2.3 Types : Equilibrium in a system implies the existenceof the followingtypes of equilibrium simultaneously,
(i) Thermal equilibrium : There is no flow of heat from one part to another i.e. T = constant.
(ii) Mechanical equilibrium : There is no flow of matter from one part to another i.e. P = constant.
(iii) Physical equilibrium : It is a state of equilibrium between the same chemical species in different
phases (solid, liquid and gaseous)
(iv) Chemical equilibrium : There is no change in composition of any part of the system with time.
4 Chemistry for JEE

2.4 Physical equilibrium


The various equilibrium which can exist in anyphysical system are,
Solid Liquid
Liquid Vapour
Solid Gas(vapour)
Solid Saturated solution of solid in a liquid
Gas(vapour) Saturated solution of gas in a liquid
(1) Solid-liquid equilibrium
Melting
H2O(s) H 2O (liquid)
Freezing

Rate of transfer of molecules from ice to water = Rate of transfer of molecules from water to ice
Rate of melting of ice = Rate of freezing of water

(2) Liquid-vapour equilibrium : When vapour of a liquid exists in equilibrium with the liquid, then
Rate of vaporisation = Rate of condensation,
Vaporisation
H 2 O (liquid) H 2 O (vapour)
Condensation

Conditions necessary for a liquid-vapour equilibrium


(i) The system must be a closed system i.e., the amount of matter in the system must remain constant.
(ii) The system must be at a constant temperature.

Y Vaporisation

Condensation
Rate

X
Time

(iii) The visible properties of the system should not change with time.

(3) Solid-vapour equilibrium : Certain solid substances on heating get converted directly into vapour
without passing through the liquid phase. This process is called sublimation. The vapour when cooled,
gives back the solid, it is called disposition.
Solid Vapour
The substances which undergo sublimation are camphor, iodine, ammonium chloride etc.
For example,Ammonium chloride when heated sublimes.
Heat
NH 4 Cl (solid) NH 4 Cl (vapour)
Cool

(4) Equilibrium between a solid and its solution : When a saturated solution is in contact with the solid
solute, there exists a dynamic equilibrium between the solid and the solution phase.
Solid substance Solution of the substance
Chemical Equilibrium 5

Example : Sugar and sugar solution. In a saturated solution, a dynamic equilibrium is established between
dissolved sugar and undissolved sugar.
Sugar (solid) Sugar (aqueous)
At the equilibrium state, the number of sugar molecules going into the solution from the solid sugar is
equal to the number of molecules precipitating out from the solution, i.e., at equilibrium,
Rate of dissolution of solid sugar = Rate of precipitation of sugar from the solution.

(5) Equilibrium between a gas and its solution in a liquid : Gases dissolve in liquids. The solubility of a
gas in any liquid depends upon the,
(i) Nature of the gas and liquid.
(ii) Temperature of the liquid.
(iii) Pressure of the gas over the surface of the solution.

2.5 Characteristics of chemical equilibrium :


(a) It is a dynamic equilibrium i.e. at this stage, reaction takes place in both the directions with same
speed so, there is no net change.
(b) At equilibrium the reaction proceeds both the side, equally
(c) At equilibrium, both reactants and products are present and their concentration do not change with
respect to time.
(d) The state of equilibrium is not effected by the presence of catalyst : It only helps to attain the
equilibrium state in less or more time.
(e) Change in pressure, temperature or concentration favours one of the reactions and thus shifts
the equilibrium point in one direction.

3. RATE OF REACTION
In a reaction , there is change in concentration of reactant or product per mole in unit time, it is known
as rate of the reaction.

  change in concentration of reactant  dc 


Rate of reaction  = –   reactant
time  dt 
Here negative sign indicate that concentration of reactants decrease with time.

change in concentration of products  dc 


Rate of reaction = + =+   product
time  dt 
Here positive sign indicate that concentration of products increase with time.

Note : The concentration change may be positive or negative but the rate of reaction is always positive.

mole / lit. mole


Unit of rate of reaction = = lit.sec = mole lit–1 sec–1
sec
For example A  B
6 Chemistry for JEE

d A 
For reactant  – [concentration decrease with time]
dt

d B
For reactant  + [concentration increase with time]
dt
[d [A], d [B] are change in concentration of A& B in time dt]

At equilibrium, since there is no net change in concentration of reactant or product.

So rate of reaction is zero.


d A  d B
– = = 0 (At equilibrium)
dt dt

4. LAW OF MASS ACTION

(a) This law was given by Guldberg and Waage.


(b) At a given temperature, the product of the concentration of products each raised to the corresponding
stoichiometric coefficients in the balanced chemical equation divided by the product of the
concentrations of the reactants raised to the corresponding stoichiometric coefficients has a constant
value.

A+BC+D

Rate of chemical reaction r  [A] [B]

r = K [A] [B]
Mathematical Expression
(i) For unitary stoichiometeric coefficients
At the constant temperature, let us consider the following reversible reaction.
A + B C + D
According to law of mass action -
Rate of forward reaction
rf  [A] [B] or rf = Kf [A] [B]
where Kf is the rate constant of the forward reaction.
Rate of backward reaction
rb  [C] [D] or rb = Kb [C] [D]
where Kb is the rate constant of the backward reaction.
At equilibrium :
Rate of reaction = Rate of forward reaction – Rate of backward reaction = 0
Chemical Equilibrium 7

Kf [A] [B] – Kb [C] [D] = 0


Kf [C] [D]
or K b = [ A] [B]

[C] [D]
or Keq =
[A] [B]

Here, k is equilibrium constant of given reversible reaction.


(ii) For non-unitary stoichiometric coefficient
n1A + n2B m1C + m2D
rf = rb

Cm Dm
1 2

Keq =
An Bn
1 2

[A], [B], [C], [D] are molar concentration of reactants and products, for dilute
solution.

Although [A] represents active mass it can be taken as Molar concentration.

IN case the reaction is not reversible as in Kinetics then stoichiometric


coefficients may not be same as the powers raised to the active masses of
reactants and products.

5. EQUILIBRIUM CONSTANTS, KC, KP , KPC


There are various methods for measuring equilibrium constant in terms of concentration, pressure, mole
fraction.
5.1 Equilibrium constant in term of concentration
Consider an equilibrium reaction as
X(g) + Y(g) Z(g)
For this reactions, which is in equilibrium, there exist an equilibrium constant (Keq) represented as
Z
Keq = X  Y 

For the given equilibrium, irrespective of the reacting species (i.e., either X + Y or Z or X + Z or Y + Z
Z
or X +Y+ Z) and their amount we start with, the ratio , X  Y  is always constant at a given temperature.
8 Chemistry for JEE

The given expression involves all variable terms (variable term means the concentration of the involved
species changes from the start of the reaction to the stage when equilibrium is reached), so the ratio
Z
X  Y  can also referred as KC.
Z
 KC = X  Y 

Thus, for the given equilibrium, it seems that Keq and KC are same but in actual practice for some other
equilibrium, theyare not same.
5.2 Equilibrium constant in terms of pressure
Assuming that the gases, X, Y and Z behave ideally.
PV = nRT
n
P= RT = CRT
T
V

P
C=
RT

PX PY PZ
 [X] = ; [Y] = and [Z] =
RT RT RT

 PZ 
  PZ  RT
 RT 
 KC = = P P
 PX   PY  X Y
   
 RT  RT 

KC PZ
= P P
RT X Y

The LHS of the above expression is a constant since KC, R and T, all are constant. This implies that RHS
is also a constant, which is represented by KP.
PZ
 KP = P  P
X Y

Thus, expression of KP involves partial pressures of all the involved species and represents the ratio of
partial pressures of product to reactants of an equilibrium reaction.
If the phase of reactant X from gaseous to pure solid. Then the equilibrium reaction can be shown as
X(s) + Y(g) Z(g)
Its equilibrium constant (Keq) would be
Z
Keq = X Y
Chemical Equilibrium 9

The concentration of X is the number of moles of X per unit volume of solid. As we known, the
concentration of all pure solids (and pure liquids) is a constant as it is represented by d/M (where d and
M represents its density and molar mass respectively). This ratio of d/M will be a constant whether X is
present initiallyor at equilibrium.
Z Z
 KC =
Keq [X] =
Y  Y 
Thus expression of KC involves only those species whose concentration changes during the reaction.
The distinction between Keq and KC is that the expression of Keq involves all the species (whether they
are pure solids, pure liquids, gases, solvents or solutions) while the KC expression involves only those
species whose concentration is a variable (like gases and solutions). Thus, expression of KC is devoid of
pure components (like pure solids and pure liquids) and solvents.

PZ
P
KC = RT  Z
PY PY
RT
PZ
Since, LHS of the expression is a constant, so the ratio P would also be a constant, represented by
Y
KP.

PZ
 KP = P
Y

5.3 Equilibrium constant in terms of both concentration & pressure


Consider the following equilibrium
X (sol.) + Y(g) Z(g) + A(g)
Z A 
KC = X  Y 

If concentration of X, Y, Z andA is expressed in terms of partial pressures

 PZ   PA 
    PZ  PA
 RT   RT 
 KC = = X P  RT
 PY 
X    Y

 RT 

PZ  PA
KC (RT) = X P
Y

The LHS of the expression is an constant (as KC, R and T all constant), which implies that the RHS will
also be a constant. But RHS of the expression can neither be called KP (as all are not partial pressure
terms) nor KC (as all are not concentration terms), so such expression that involves partial pressure and
concentration terms both are reffered as KPC.
10 Chemistry for JEE

PZ  PA
 KPC = XPY
Thus, KP can exist only for that equilibrium which satisfies these two conditions.
(a) At least one of the reactant or product should be in gaseous phase and
(b) No component of the equilibrium should be in solution phase (Because when solution is present, the
equilibrium constant would be called KPC)
5.4 Equilibr ium constant in ter ms of moles fr action (K X) :

Kx is not true equilibrium constant for all the reactions.

A + B C + D
Mole fraction at equilibrium XA XB XC XD
XC  XD
KX 
XA  XB

It is not necessary that all the types of equilibrium constants are defined for
every reaction for eg. for a reaction involving only solutions Kp is not defined.

Can you guess what will be the equilibrium constant for solid - liquid equilibria.

6. CHARACTERISTICS OF EQUILIBRIUM CONSTANT


(i) The expression for equilibrium constant, K is applicable only when concentrations of the reactants and
products have attained their equilibrium values and do not change with time.
(ii) The value of equilibrium constant is independent of initial concentration of the reactants and product.
(iii) Equilibrium constant has one unique value for a particular reaction represented by a balanced equation at
a given temperature.
(iv) The equilibrium constant for the reverse reaction is equal to the inverse of the equilibrium constant for the
forward reaction.
H2(g) + I2(g) 2HI(g)
PHI2
Kp = P .P
H 2 I2

2HI(g) H2(g) + I2(g)


PH .PI 1
Kp' =
2
2
2

PHI Kp
1
KP' = K
P
(v) The equilibrium constant K, for a reaction is related to the equilibrium constant of the corresponding
reaction whose equation is obtained by multiplying or dividing the equation.
H2(g) + I2(g) 2HI(g)
Chemical Equilibrium 11

PHI2
Kp = P .P
H2 I2

1 1
H2(g) + I2(g) HI(g)
2 2
PHI
KP" = P1/ 2 .P1/ 2 = Kp
H2 I2

(vi) If reaction is performed in multiple steps


A B ; overall reaction
Step-1 : A(g) C(g) + D(g) K p1
Step-2 : C(g) E(g) K p2

Step-3 : D(g) + E(g) B(g) K p3

A(g) B(g) Kp then Kp = K p1 . K p 2 . K p3

Factors which do not influence equilibrium constant :

(a) Concentration of reactants and products.


(b) Pressure and volume.
(c) Presence of catalyst.
(d) Addition of the inert gas at constant Pressure and volume.

Factors which influence the equilibrium constant :


A. Mode of representation of chemical reaction.
B. Stoichiometry of reaction.
C. Temperature.

A. Mode of representation of reaction –


If we take reaction 2HI H2 + I2
Then, we write the value of equilibrium constant K C1 for the above reaction as following.
[H2 ][I 2 ]
K C1 = ....... (i)
[HI] 2

Now, if we take reaction H2 + I2 2HI


Then, we write the value of equilibrium constant K C2 for above reaction as following
[HI] 2 1
K C2 =  ...... (ii)
[H 2 ][I 2 ] K C1
B. Stoichiometry of the reaction –
Method of writing the equation of the reversible reaction is called as stoichiometry of the reaction.
Now, we consider the formation of HI(g) by the combination of H2(g) and 2(g).
H2 +  2 2H
12 Chemistry for JEE

The expression of its equilibrium constant is-


[H]2
K C1
= [H ][ ]
2 2

If the equation of above reaction is written by following method –


1 1
H2(g) +  (g) H(g)
2 2 2
The expression for the equilibrium constant is –
[H]
K C2
= [H ] ½ [ ] ½
2 2

on the basis of comparing both the equilibrium constant equation.


K C2 = K C1 or ( K C 1 )1/2

When we divide a reaction by a factor 'n' in the equation, the value of new
equilibrium constant is equal to the root of n of the previous equilibrium
constant.

For Example – Suppose, the equilibrium constant for the following reaction.
A + B C + D is K1 then
for the reaction
1 1 1 1
A+ B C+ D
n n n n

the value of the equilibrium constant K2 is equal to n K 1 or (K1)1/n.

K2 = K11/ n

C. Temperature –
Increase in temperature favours the endothermic reaction and decrease in temperature favours the
exothermic reaction for the forward reaction so for exothermic reactions, the value of Kc and Kp
decrease with rise in temperature while for endothermic reactions, the value of Kc and Kp increases
with rise in temperature. This type of variation in equilibrium constant with temperature given by
Van't Hoff equation as follows -

H LM 1  1 OP
log K2 – log K1 =
2.303R NT T Q
1 2

K2 H LM 1  1 OP
or log K =
1 2.303R NT T Q
1 2
(Refer Page No. 24-25)
Where,
K2 = equilibrium constant at temperature T2
K1 = equilibrium constant at temperature T1
H = Energy of reaction of constant temperature
R = Molar gas constant
Chemical Equilibrium 13

According to the temperature, reaction are of three types.

(a) Non-thermic reaction means H = 0


log K2 – log K1 = 0
log K2 = log K1
There is no effect of temperature on this type of reaction.
(b) Endothermic reaction  H = ( + ) ve
log K2 – log K1 = ( + ) ve,
means K2 > K1
On increasing of temperature, equilibrium constant will also increase for this
type of reaction.
(c) Exothermic reaction  H = ( – ) ve
log K2 – log K1 = ( – ) ve,
means K2 < K1
On the increase of temperature equilibrium constant will decreases for exothermic
reaction.

Units of KC and Kp
The concentration is expressed in the term of moles per litre. Therefore, units of Kc will be (moles
litre–1)n.
In the same way, partial pressure are measured by the unit of atmospheres and therefore units of
Kp will be (Atmospheres)n.

Value of n Unit of Kc Unit of Kp

0 No unit No unit
>0 –1
(Moles l ) n (atm)ng
<0 (Moles l–1)n (atm)ng

Relation between Kp and KC


Let us consider the following reaction
n1 A + n2 B m1 C + m2 D.
The value of Kc for the reaction is,
[C] m1 [D] m2
Kc =
[A] n1 [B] n2
According to gas law PV = n RT
FG n IJ
P= H vK RT
T ....... (1)

Here n
= no. of moles = [ ] = Active mass
v lit.
14 Chemistry for JEE

(p C ) m1 (p D ) m2
 KP =
(p A ) n1 (p B ) n2

on putting the value of 'p' in the formula of KP by the equation (1)


([C] RT) m1 ([D]RT) m2
KP =
([A]RT) n1 ([B]RT) n2

[C] m1 [D] m2 (RT) m1 m2


KP =
[A] n1 [B] n2 (RT) n1 n2

KP = KC (RT) (m1 m2 )(n1 n2 )


n g
KP = K C RT 
n = (m1 + m2) – (n1 + n2)
ng = number of moles of gaseous products – number of moles of gaseous reactants.
T = Absolute temperature.

Units of R should be taken in accordance with the units of Kp and Kc.

Illustration
1. At 700 K, the equilibrium constant KP, for the reaction
2SO3(g) 2SO2 (g) + O2 (g)
–3
is 1.8 × 10 kPa. What is the numerical value of KC for this reaction at the same temperature–
(A) 3.09 × 10–7 mole litre–1 (B) 9.03 × 10–7 mole litre–1
(C) 5.05 × 10–9 mole litre–1 (D) 5.05 × 10–5 mole litre–1
Ans. A
Sol. We know the relationship
KP = KC(RT)n
Here KP = 1.80 × 10–3
1.8  10 3
Kp = atm
101.3
= 1.78 × 10–5 atm
R = 0.0821 litre atm K–1 mol–1
n = 3 – 2 = 1
T = 700 K
KP 1.78  10 5
KC = (RT) n =
0.0821 700
= 3.09 × 10–7 mole litre–1.
Chemical Equilibrium 15

7. APPLICATIONS OF EQUILIBRIUM CONSTANT


Consider some applications of equilibrium constant and use it to answer question like:
(i) predicting the extent of a reaction on the basis of its magnitude.
(ii) predicting the direction of the reaction.
7.1 Predicting the extent of a reaction
The magnitude of equilibrium constant is veryuseful especially in reactions of industrial importance.An
equilibrium constant tells us whether wecanexpect a reaction mixtureto contain a highor lowconcentration
of product(s) at equilibrium. (It is important to note that an equilibrium constant tells us nothing about the
rate at which equilibrium is reached). In the expression of KC or KP, product of the concentrations of
products is written in numerator and the product of theconcentrations of reactants is written in denominator.
High value of equilibrium constant indicates that product(s) concentration is high and its low value
indicates that concentration of the product(s) in equilibrium mixture is low.
H2 (g) + Br2(g)  2HBr(g)

(PHBr ) 2
KP = 18
(PH )(PBr ) = 5.4 × 10
2 2

The large value of equilibrium constant indicates that concentration of the product, HBr is very high and
reaction goes nearly to completion.
Similarly, equilibrium constant for H2(g) + Cl2(g) 2HCl(g)
is very high and reaction goes virtually to completion.

[HCl]2
KC = = 4.0 × 1031
[H 2 ][Cl 2 ]

Thus, large value of KP or KC (larger than about 103), favour the products
strongly. For intermediated values of K (approximately in the range of 10–3 to
103), the concentrations of reactants and products are comparable. Small values
of equilibrium constant (smaller than 10–3), favour the reactants strongly.

At 298 K for reaction, N2(g) + O2(g)  2NO(g)

[NO]2
KC = = 4.8 × 10–31
[N 2 ][O 2 ]
The very small value of KC implies that reactants N2 and O2 will be the predominant species in the
reaction mixture at equilibrium.

7.2 Predicting the direction of the reaction.


The equilibrium constant is also used to find in which direction the reaction will proceed for a given
concentration of reactants and products. For this purpose, we calculate the Reaction Quotient (Q). The
reaction quotient is defined in the same way as the equilibrium constant ( with molar concentrations to
give QC, or with partial pressure to give QP) at any stage of reaction. For a general reaction:
n1 A + n2 B m1 C + m2 D.
16 Chemistry for JEE

m m2
[C] 1 [D]
QC = n n2
[A] 1 [B]
Then, if QC > Kc , the reaction will proceed in the backward direction
if QC < Kc, the reaction will move in the forward direction
if QC = Kc, the reaction mixture is already at equilibrium.
In the reaction, H2(g) + I2(g)  2Hl(g), if the molar concentrations of H2 , I2 and HI are 0.1 mol L–1
respectively at 783 K, then reaction quotient at this stage of the reaction is

[HI]2 (0.4) 2
QC = = =8
[H 2 ][I 2 ] (0.1)(0.2)
KC for this reaction at 783 K is 46 and we find that QC < KC. The reaction, therefore, will move to right
i.e. more H2(g) and I2(g) will react to form more HI (g) and their concentration will decrease till
QC = KC.

8. DEGREE OF DISSOCIATION
Degree of dissociation is the fraction of a mole of the reactant that underwent dissociation. It is represented
by ''
no. of moles of reactant dissociated
 = no. of moles of reactant present initially

For example,
Let the equilibrium reaction is the dissociation equilibrium of NH3 into N2 and H2.
1 3
NH3 (g) N2 (g) + H2(g)
2 2
Moles initially a 0 0
a 3a
Moles at equilibrium a(1–)
2 2
Here,  represented the degree of dissociation.

Illustration
2. 2.56 gm of sulphur S8 (s) is taken which is in equilibrium with its vapour according to reaction,
S8(s)  S (g)
if vapours occupies 960 m at 1 atm & 273 K then the degree of dissociation of S8(s) will be
[Given : R = 0.08]
(A) 0.5 (B) 0.55 (C) 0.4 (D) 0.44
Ans. B
2.56
Sol. n S8 = = 0.01
8 32
S8(s)  8 S (g)
0.01(1–) 8 × 0.01 × 
PV = nRT
960
1×  (0.018 )  0.08 273
1000
 = 0.55
Chemical Equilibrium 17

9. CALCULATION OF KP & KC
(a) Homogeneous equilibrium in gaseous phase
(b) Homogeneous equilibrium in solution phase
(c) Equilibrium constant for various heterogeneous equilibrium

9.1 Homogeneous equilibrium in gaseous phase


Formation of Nitric Oxide : (n = 0)
Calculation of KC : –
Suppose the initial concentration of N2 and O2 is a and b respectively. x is the degree of dissociation.
N2 + O2 2NO
Initial moles a b 0
moles at equilibrium (a–x) (b–x) 2x
(a  x) (b  x) 2x
Active mass (mol l–1)
V V V
Here, V is the volume of container in litre.
According to the law of mass action
[NO] 2
KC = [N ][O ]
2 2

FG 2x IJ2

H vK
Substituting the values in the above equation KC =
FG a  x IJ FG b  x IJ
H v KH v K
4 x2
KC =
(a – x) (b – x)

Kc for this reaction is independent of V of the reaction container.


Calculation of KP :
All the things being same as above, except pressure. Let P atmosphere is the pressure at equilibrium.
N2 + O2 2 NO
Initial moles a b 0
moles at equilibrium (a–x) (b–x) 2x
Total no. of moles = (a–x) + (b–x) + 2x = (a+b)
The partial pressure of the above three species can be calculated as below-
(a  x)P
PN =
2 (a  b)
(b  x) P
PO2 = ( a  b)
(2x) P
PNO = (a  b)
According to the law of mass action
[PNO ] 2
KP = [PN2 ][PO 2 ]
18 Chemistry for JEE

substituting the value of PNO , PN2 , PO2 in the above equation of KP -

LM (2x)P OP 2

N (a  b) Q
KP =
LM (a  x)P OP LM (b  x)P OP
N (a  b) Q N (a  b) Q
4x 2
KP = (a  x)(b  x)

9.2 Thermal Dissociation of Phosphorus pentachloride- (n > 0)


A. Calculation of KC - Suppose one mole of PCl5 is take in a closed container of V litre.
Further at equilibrium x mol of PCl5 dissociated
PCl5 PCl3 + Cl2
Initial moles 1 0 0
moles at equilibrium (1–x) x x
1 x x x
Concentration (mol l–1)
v v v
According to law of mass action
[PCl 3 ][Cl 2 ]
KC = [PCl 5 ]
Substituting the values in the above equation.
FG x IJ FG x IJ
H vK H vK
KC = FG 1  x IJ
H vK

The formula of KC has V in the denominator, hence the equilibrium will be affected by V of the
reaction container for the given reaction.
If x << 1 then 1 – x  1
x2
So, KC =
v
x2 = KC .v
x2  v
x v

If we increase the volume, the dissociation x is also increased.


B. Calculation of KP -
PCl5 PCl3 + Cl2
initial moles 1 0 0
moles at equilibrium 1–x x x
Chemical Equilibrium 19

Total no. of moles at equilibrium,


(1 – x) + x + x = (1 + x) moles
According to law of mass action
PPCl3  PCl2
KP = PPCl5
xP
At equilibrium PPCl3 = (1  x)
xP
PCl2 =
(1  x)
(1  x)P
PPCl5 =
(1  x)
Substituting the values in the above equation of KP -
FG x  P IJ FG x  P IJ
KP =
H 1 x K H 1 x K
(1  x)  P
(1  x)
x 2P
KP =
1  x2
The equation of KP is not independent of pressure.
suppose, x << 1 then 1 – x2  1

KP = x2 P
KP
x2 =
P
1
x2 
P
1
x
P

The degree of dissociation of PCl5 is inversely proportional to the square root


of pressure so, decrease of pressure increases dissociation of PCl5.

9.3 Formation of Ammonia – (n < 0)


A. Calculation of KC : –
N2 + 3H2 2NH3
Initial moles 1 3 0
moles at equilibrium (1–x) (3–3x) 2x
FG 1  x IJ FG 3  3x IJ  2x 
Active mass (mol l–1) H vK H v K 
 v 

According to law of mass action


[NH 3 ] 2
KC =
[N 2 ][H 2 ] 3
20 Chemistry for JEE

Substituting the values in the above equation-


FG 2x IJ 2

H vK
KC = FG 1  x IJ FG 3  3x IJ 3

H v KH v K
4x 2 v 2
KC =
(1  x)(3  3x) 3

4x 2 v 2
KC 
27(1– x)4
The formula of KC has V in the numerator, hence the equilibrium will be affected by V of the
reaction container.
Dependence If, x << 1 then, (1 – x)4 = 1
4x2 v 2
KC =
27

27K C
x2 =
4V 2
1 1
x2  x
V2 V
If we increase the volume of the container the degree of dissociation x is decreased.

B. Calculation of KP :
N2 + 3H2 2NH3
Initial concentration 1 3 0
moles at equilibrium (1 – x) (3 – 3x) 2x
Total number of moles at equilibrium
= (1 – x) + (3 – 3x) + 2x = (4 – 2x)

According to the law of mass action


(PNH3 ) 2
KP = (P )  (P ) 3
N2 H2

(2x). P
At equilibrium PNH3 =
(4  2x)

(1  x). P
PN2 =
( 4  2x)

(3  3 x).P
PH2 =
(4  2x)

Substituting the values in the above equation of KP.


Chemical Equilibrium 21

FG 2x .PIJ 2

H 4  2x K
KP = FG 1  x .PIJ FG 3  3x .PIJ 3

H 4  2x K H 4  2 x K
4x 2 4  2x 2
KP =
1 x  3  3x 3 p 2
16x 2 (2 – x)2
KP 
27(1– x) 4 P 2
The equation of KP is not independent of pressure
suppose, x << 1 then,
(1 –x)4 = 1
and (2 – x)2 = 4
64 x 2
KP =
27P 2
x 2   P2
xP

If we increase the pressure the degree of dissociation x is also increased.


Illustration
3. At a certain temperature (T), the equilibrium constant (KC) is 1 for the reaction
N2(g) + 3H2(g)  2NH3(g)
If 2 moles of N2, 4 moles of H2 , 6 moles of NH3 & 3 moles of inert gas are introduced into a two litre
rigid vessel at constant temperature T. It has been found that equilibrium concentration of H2 & NH3 are
equal then what is the equilibrium concentration of N2 (in M)?
2
6
 
2
Sol. QC = 3  1.125
 2  4 
  
 2  2 
 QC > KC so reaction will proceed in backward direction.

N2(g) + 3H2 (g)  2NH3 ;


at equilibrium : 2 + x 4 + 3x 6 – 2x
 At equilibrium [H2(g) ] = [NH3(g)]

4  3x 6  2x
= x = 0.4
2 2
2 x
 [N2(g) ] at equilibrium =  1.2 M
2
22 Chemistry for JEE

9.4 Homogeneous equilibrium in solution phase


Formation of ethyl acetate
Equilibrium is represented as
C2H5(OH)(l) + CH3COOH(l) CH3COOC2H5(l) + H2O(l)
Initial moles 1 1 0 0
Moles at equilibrium 1–x 1–x x x
1 x 1 x x x
Active mass (mol l–1)
V V V V

CH3COOC2H5  H 2O x x
 x2
V V
KC = C2H5OHCH3COOH KC = 1  x  1  x  =

1 x  1 x 
V V

Illustration
4. Determine the amount of ester present under equilibrium when 3 moles of ethyl alcohol react with 1 mole
of acetic acid, when equilibrium constant of the reaction is 4.
Sol. CH3COOH + C2H5OH CH3COOC2H5 + H2O

1 x 3 x x x
V V V V
xx
  
V V
KC = 4 =  1 x   3  x 
  
 V  V 
3x2 – 16x + 12 = 0
x = 0.903
Amount of ester at equilibrium = 0.903 mole

9.5 Equilibrium constant for various heterogeneous equilibrium


Heterogenous equilibrium results from a reversible reaction involving reactants and product that are in
different phases. The law of mass action is applicable to a homogeneous equilibrium and is also applicable
to a heterogeneous system.
Decomposition of solid CaCO3 into solid CaO and gaseous CO2
Let ‘a’ moles of CaCO3 are taken in a vessels of volumer ‘V’ litre at temperature ‘T’ K.
CaCO3(s) CaO(s) + CO2(g)
Moles initially a 0 0
Moles at equilibrium a–x x x
Chemical Equilibrium 23

CaO  CO 2 
Keq = CaCO 3 
As CaCO3 and CaO(s) are pure solids, so their concentration is unity
x
 KC = [CO2] = .....................(1)
V
Assuming CO2 gas to behave ideallyat the temperature & pressure of the reaction, the molar concentration
PCO
2
of CO2 can be written using ideal gas equation as
RT

PCO
 KC = 2
RT

KC(RT) = PCO
2

Since KC, R and T are constant, their product will also be a constant referred as KP.
xRT
 KP = PCO 2 = ....................(2)
V
Illustration
5. At 87°C, the following equilibrium is established:
H2(g) + S(s)  H2S(g), Kc = 8 × 10–2
If 0.3 mole hydrogen and 2 mole sulphur are heated to 87°C in a 2 L vessel, what will be the partial
pressure of H2S approximately at equilibrium. [Use R = 0.08 atm.L / mol. K]
(A) 0.32 atm (B) 0.43 atm (C) 0.62 atm (D) 4.0 atm
Ans. A
[H 2S(g)] x
Sol. KC = [H (g)]  8 × 10–2 =
2 0.3  x
0.024 – 0.08 = x
0.024 = 1.08 x
0.022 0.08 360
x = 0.022 PH
2S
=   0.32 atm
2

10. MULTIPLE EQUILIBRIUM


In multiple equilibrium the product molecules (s) in one equilibrium system are involved in a second
equilibrium process.

KC 
C B
A(g) + B(g) C(g) + D(g) 1 A B

KC 
F G 
C(g) + E(g) F(g) + G(g) 2 C E 
24 Chemistry for JEE

KC 
D F G 
Overall reaction : A(g) + B(g) + E(g) D(g) + F(g) + G(g) 3 A  B E
In this case, one of the product molecule, C(g) of the first equilibrium reaction combines with E(g) to give
F(g) and G(g) in another equilibrium reaction, so in the overall, C(g) will not appear on either side.
 
The equilibrium constant K C3 of the overall reaction can be obtained if we take the product of the

expression of K  and K .
C1 C2

C D F G  D F G 


KC × KC =
1 2 A  B × C E  = A  B E 
 KC × KC = KC
1 2 3

If a equilibrium reaction can be expressed as the sum of two or more equilibrium reactions, the equilibrium
constant for the overall reaction is given by the product of the equilibrium constant of the individual
reactions.

11. SIMULTANEOUS EQUILIBRIUM


In simultaneous equilibrium more than one equilibrium are established in a vessel at the same time and
any one of the reactant or product is common in more than one equilibrium, then the equilibrium
concentration of the common species in all the equilibrium would be same.
For example, if we take CaCO3(s) and C(s) together in a vessel of capacity ‘V’ litre and heat it at
temperature ‘T’ K, then CaCO3 decomposes to CaO(s) and CO2(g). Further, evolved CO2 combines
with the C(s) to give carbon monoxide. Let the moles of CaCO3 and carbon taken initially be ‘a’ and ‘b’
respectively.
CaCO3(s) CaO(s) + CO2(g)
Moles at equilibrium a-x x (x–y)
CO2(g) + C(s) 2CO(g)
Moles at equilibrium (x–y) (b–y) 2y

Thus, as CO2 is common in both the equilibrium so its concentration is same in both the equilibrium
constant expression.
xy
Equilibrium constant for first equilibrium, K C1 = [CO2] =
V

CO 2 2y 2 V  4y 2
Equilibrium constant for second equilibrium, K C 2 =
CO 2  = V 2 x  y  Vx  y 
12. EQUILIBRIUM CONSTANTAS PER KINETICS
According to the kinetic theory of gases, in any gaseous system, different gas molecules travel with
different speeds. The molecular collision with low energy can never cause bond cleavage and hence
can not result the product formation. Only those molecular collision result the formation of product in
which the molecules collides with a certain minimum energy.
Threshold energy -The minimum amount of energy, which the colliding molecules must posses in order
to make the chemical reaction to occur, is known as Threshold energy, Et.
Chemical Equilibrium 25

Activation energy -The minimum amount of energy required to make active participated of almost all
molecules in a reaction is calledActivation energy, Ea. The activation energy is equal to Et – ER, where
ER is the average energy level of reactant molecules.
Activation energy for forward reaction = Threshold energy – Potential energy of reactants
The activation energy of reaction depends on the nature of reactant and temperature. It decreases with
increase in temperature but the decrease is so small that it is normallyconsidered temperature independent.
Et = Threshold energy
Et
Ea =Activation energy of forward reaction Ea Eb
Eb= activation energy of backward reaction
P1 = Potential energy of reactants

Energy
Et
P2 = Potential energy of products Reactants
Product
Kf P1
A(g) B (g) + C(g) P2
Kb
Reaction Co-ordinates
 d[A]
= Kf [A] – Kb [B] [C]
dt
 d[A]
At equilibrium =0
dt
K f [B][C]
  Kc
Kb [A]
According toArrhenius equation
where k = A · e Ea / RT ; A : pre-exponential factor
 E a (f ) / RT
kf = Af · e Ea : activation energy
 E a (b) / RT
kb = Ab · e
E / RT
kf A f e a (f )
K= k = E / RT
b A b e a (b)

 Af 
k =A· e–H/RT A  
 A b 

where H = Ea(f) – Ea(b)


H
ln K1 = ln A – RT (at temp T1, K = K1)
1

H
ln K2 = ln A – RT (at temp T2, K = K2)
2

K 2 H  1 1 
ln    
K1 R  T1 T2 
26 Chemistry for JEE

13. CALCULATION OF DEGREE OF DISSOCIATION BY VAPOUR DENSITY


MEASUREMENT
Reactions in which there is a change in the number of moles after dissociation, the extent of
dissociation can be determined by vapour density measurement.
Consider the following reaction -
PCl5 PCl3 + Cl2
Initially 1 0 0
moles at (1 – )  
equilibrium ('' is the degree of dissociation )
Total number of moles at equilibrium = (1 – ) +  +  = (1 + )
V is the volume occupied by 1 mol of PCl5(s) which have vapour density is 'D' before dissociation
and after dissociation is 'd'. Under the same conditions, the volume occupied by (1 + ) moles at
equilibrium would be (1 + ) V litre.
1
Density 
Volume

1 1
D d  (1  )V
V

1
D V D Dd
or = 1 = (1 + ) or  = 1 =
d d d
(1  )V

Molecular mass = 2 × Vapour density


M t  Mo
so  = Mo
where Mt = calculate molecular mass
Mo = observed molecular mass

Note : When one mole of reactant on dissociation gives 'n' moles of gaseous products.
A nB
Initial moles 1 0

Moles after dissociation 1– n

Total moles of equilibrium 1 – + n = 1 + (n –1)

 1n 1 
D
d

Dd 1 D  d 
 n 1  or  = n 1 d
d
Chemical Equilibrium 27

Illustration
6. N2O3 on decomposition gives NO and NO2, they are found to be in equilibrium at 300 K. If the vapour
density of such an equilibrium mixture is 23.75, calculate percentage bymass of N2O3 in the equilibrium
mixture?
(A) 80 % (B) 60 % (C) 40 % (D) 20 %
Ans. C
Sol. N2O3  NO + NO2
1 –  

Dd 38  23.75
=  = 0.6
d(n 1) 23.75(2 1)

wt.of N 2 O 3
Mass % of N2O3 in the equilibrium mixture =  100
Total wt.

0.4  76
=  100 = 40 %
0.6  30  0.6  46  0.4 76

14. GIBB'S FREE ENERGYAND EQUILIBRIUM CONSTANT


Gibb's free energy (G) of a system is defined as the thermodynamic quantity of the system, the decrease
in whose value during a process is equal to useful work done by the system.
Standard free energy change is defined as the free energy change for a process at 298 K and 1 atm
pressure in which the reactants in their standard state are converted to products in their standard state.
It is denoted as Gº.

Note : Standard free energy change (Gº) is not the free energy change at equilibrium.
Gº is related to K (equilibrium constant) by the relation
Gº = – RTln K.
Gº = – 2.303 RT log K.
K may either be KC or KP.
The units of Gº depends only on RT. T is always in Kelvin, and if R is in Joules, Gº will be in joules
and if R is calories then Gº will be in calories.

Illustration
7. NO and Br2 at initial partial pressures of 98.4 and 41.3 torr, respectively, were allowed to react at
300K. At equilibrium the total pressure was 110.5 torr. Calculate the value of the equilibrium constant
and the standard free energy change at 300 K for the reaction 2NO(g) + Br2(g) 2 NOBr(g).
Sol. 2NO(g) + Br2(g) 2NOBr (g)
Initial pressure 98.4 41.3 0
x
At equilibrium 98.4-x 41.3- x
2
28 Chemistry for JEE

Total pressure at equilibrium is 110.5 torr


x
 98.4-x + 41.3- + x = 110.5
2
x = 58.4 torr
Now, 1 atm = 760 torr; x = 7.68 × 10–2 atm

PNOBr = 7.68 × 10–2 atm; pNO = 98.4-x = 40 torr = 5.26 × 10–2 atm
x
PBr2 = 41.3 - = 12.1 torr = 1.59 × 10–2 atm
2

p NOBr 
 2
7.6810  2 2

p  5.2610  1.5910 


Kp = = 2 = 134 atm–1
pNO 2
Br2
2 2

Gº = – 2.303 RT log K = – 2.303 (1.99) × 10–3 (300) (log 134)

= – 2.92 k cal = 12.2 k J.


[If R is used as 1.99 cal/mol K, then Gº will be in cal. If R is used as 8.314 J/mol K, then Gº will be
in joules. But KP must be in (atm)n]

15. LE CHATELIER PRINCIPLE


Chemical equlibrium represents a balance between forward and reverse reactions. In most cases, this
balance is quite delicate. Changes in concentration, pressure, volume and temperature may disturb the
balance and shift the equilibrium position so that more or less of the desired product is formed. There is
a general rule (named Le Chaterlier principle) that helps us to predict the direction in which an equilibrium
reaction will move when a change in concentration, pressure, volume or temperature occurs. Le Chatelier’s
principle state that if an external stress is applied to a system at equilibrium, the system adjusts in such a
way that the stress is partially offset.

The word “stress” here implies a change in concentration, pressure, volume, addition of an inert gas or
temperature that removes a system from the equilibrium state.
Le Chatelier principle can be explained using the following equilibrium reaction
PCl5(g) PCl3(g) + Cl2(g)
Let the moles of PCl5, PCl3 and Cl2 at equilibrium be a, b and c respectively,Also let the volume of the
container in which equilibrium is established be ‘V’ litre and the total pressure of the system at equilibrium
be PT atm.

P  P 
PCl3 Cl 2


b
abc

 PT  
c
 abc

 PT 

K =
P
P  PCl5
=


a 
 PT 
abc 
Chemical Equilibrium 29

bc  PT

a a  b  c 
KP = .............(1)

The total pressure of the system (PT) can be given as (assuming all gases at equilibrium behave ideally
under the given conditions)
a  b  c RT
PT =
V
PT RT
 
a  b  c  V
PT
Inserting the value of
a  b  c  in equation (i), we get
bc  RT
KP = ..............(2)
aV
Let us examine the effect of change of certain parameters like moles of reactant, moles of product,
volume, temperature, addition of inert gas and addition of catalyst on the given equilibrium.

15.1 Change in number of moles of reactant


If we add ‘d’ moles of PCl5 to the equilibrium mixture, the equilibrium would be disturbed and the
bc  RT
expression
a  d V becomes QP.As QP < KP, so the net reaction moves in the forward direction till QP
becomes equal to KP.
Thus for any equilibrium, when more reactant is added to (or some product is removed from) an
equilibrium mixture, the net reaction moves in the forward direction (as Q < K) to establish a new
equilibrium state.

15.2 Change in number of moles of product


Let ‘d’ moles of PCl3 (or Cl2) are added to the equilibrium. The equilibrium would be under stress and
b  d c  RT
thus the expression would become QP. Since QP > KP, so the net reaction moves in the
aV
reverse direction till QP becomes same as KP.
Thus for any equilibrium, when product is added to (or some reactant is removed from) an equilibrium
mixture, the net reaction moves in the reverse (backward) direction (as Q > K) to establish a new
equilibrium state.

15.3 Change in volume


Let the volume of the container be increased from V to V' litre. The equilibrium would be disturbed and
bc  RT
the expression becomes QP. The value of QP is less than KP, so the net reaction moves in the
a  V'
forward direction to establish new equilibrium. But when the volume of the container is decreased, the
reaction moves in the backward direction to again attain the equilibrium state.
30 Chemistry for JEE

Thus for anyequilibrium, on increasing the volume of the container, the net reaction shifts in the direction
of more moles of the gases while on decreasing the volume of the vessel, the reaction goes in the
direction of fewer moles of the gases.

15.4 Addition of an inert gas


The effect of addition of an inert gas can be studied under two conditions (i), at constant volume (ii) at
constant pressure.
(i) At constant volume
Let ‘d’ moles of an inert gas are added to the equilibrium mixture at constant volume. The total number
of moles of the system increases so is the pressure of the system but the partial pressure of all the species
PT' RT
would still be same. Let the total pressure becomes PT’then
a  b  c  d  =
V
. As R, T and V are

bc  RT
constant, so the expression would still be equal to KP. As, QP = KP, then net reaction does
aV
not move at all.
Thus for any equilibrium when an inert gas is added at constant volume, the equilibrium remains
unaffected whether the equilibrium reactions have n equal to zero or non-zero.
(ii) At constant pressure
Now, let ‘d’moles of an inert gas are added to the equilibrium mixture at constant pressure to keep the
pressure constant, volume of the vessel should increases. Let the volume of the vessel increases from V
bc  RT
to V’litre. So, the expression becomes QP.As the value of QP < KP, so the net reaction moves
a  V'
in the forward direction to establish new equilibrium state. Thus, addition of an inert gas at constant
pressure has the same effect as produced by the increased volume of the container.
Thus, for equilibrium having n = 0, when an inert gas is added at constant pressure, the equilibrium
remains unaffected (since V does not appear in the expression of KP) while for equilibrium having
n  0, the addition of an inert gas at constant pressure causes reaction to move in the direction of more
moles of the gases.
15.5 Addition of a catalyst
Acatalyst enhances the rate of a reaction by lowering the reactions’activation energy.Actually a catalyst
lowers the activation energy of the forward reaction and the reverse reaction to the same extent, so the
presence of a catalyst does not alter the equilibrium constant nor does it shift the position of an
equilibrium system.Adding a catalyst to a reaction mixture that is not at equilibrium will simply cause
the mixture to reach equilibrium faster.
15.6 Change in temperature
If KP increases, the net reaction moves forward while if KP decreases, the net reaction moves backward.
The variation of KP with temperature is given byVan’t Hoff equation as

KT
H  1 1 
   where T > T
2
log K =
T1 2.303R  T1 T2  2 1
Chemical Equilibrium 31

All reactions are either endothermic or exothermic in nature. For an endothermic reactions, H is positive
and with an increase in temperature of the system to T2 K from T1 K, the RHS of the expression
 
becomes positive. Thus, equilibrium constant at higher temperature K T2 would be more than the

 
equilibrium constant at lower temperature K T1

But for an exothermic reaction, H is negative and on increasing the temperature of the system from
T1 K to T2 K, the RHS of the expression becomes negative. So the equilibrium constant at higher
temperature would be less than equilibrium constant at lower temperature.
The give equilibrium, PCl5(g) PCl3(g) + Cl2(g) is endothermic in nature. So, with the increase of
temperature from T1 K to T2 K, KP and QP both increases. Therefore, equilibrium shifts in the forward
direction.
Thus, for an endothermic reaction (H = positive), with the increase of temperature, net reaction moves
in the forward direction and the decreases in temperature favours backward reaction while for an
exothermic reaction (H = negative), net reaction moves in the backward direction with the increase of
temperature and in forward direction with the decrease temperature.
In general, with the increases of temperature, net reaction moves in that direction where the heat
is absorbed and the effect of increasing temperature is nullified.

15.7 Change in more than one parameter


For the given equilibrium, if the number of moles of PCl3 is increased four folds and the volume of the
4b  c  RT
vessel is doubled, then the equilibrium would be disturbed. The expression would becomes
a  2V
QP. Since QP > KP, so the net reaction moves in reverse direction till QP becomes equal to KP.
Thus, when two or more parameters are simultaneously changed for any equilibrium, find the
changed value of Q and K and compare them. If Q = K, there will be no effect on the reaction, if Q
> K, the net reaction moves in the backward direction. While if Q < K, net reaction moves in the
forward direction.

15.8 Application of Le chatelier principle on physical equilibrium


A. Melting of Ice :
H2O(s) H2O ()
Ice water
more volume less volume
If we increase the pressure, the equilibrium will in the direction of less volume. Hence, the rise of
pressure, more ice will melt into water i.e. melting point of ice is decreased by rise of pressure.
32 Chemistry for JEE

B. Vaporization of liquid -
H2 O() H2O(g)
water vapour
less volume more volume

Vaporization of a liquid is endothermic process in the nature i.e. the evaporation of a liquid into
its vapour is completed by absorption of heat, so the rise of temperature will favour vaporization.
On the other hand in this process, on increase of pressure the equilibrium will shift in the direction
of less volume means water cannot be converted into vapour and boiling point increases.

C. Melting of Sulphur :
Sulphur(s) Sulphur()
less volume more volume
On increase in pressure, the equilibrium will shift towards less volume means solid is not converted
into liquid and thus, melting point of sulphur increases.

Illustration
8w. Following two equilibrium is simultaneouslyestablished in a container
PCl5(g) PCl3(g) + Cl2(g)
CO(g) + Cl2(g) COCl2(g)
If some Ni(s) is introduced in the container forming Ni (CO)4 (g) then at new equilibrium
(A) PCl3 concentration will increase
(B) PCl3 concentration will decrease
(C) Cl2 concentration will remain same
(D) CO concentration will remain same
Ans. B
KP & KC for different reactions
Chemical Equilibrium
33
34 Chemistry for JEE

THE ATLAS
Chemical Equilibrium 35

EXERCISE-1 (Subjective Questions)


Chemical Equilibrium and Equilibrium Constant
Q.1 Which of the following statement is True / False for dynamic equilibrium?
(a) concentration of reactant and products are always equal.
(b) rate of forward and backward reaction are equal
(c) ratio of concentration of reactants and products is constant
(d) ratio of partial pressure of reactants and products is constant.
(e) vapour density of mixture is constant.
(f) concentration of reactants and products are constant.
(g) The equilibrium constant expression depends only on the stoichiometry of the reaction, not on the
mechanism.

Q.2 Write expression for KP, KC and KPC (whichever is applicable) :


(a) 2O3(g)  3O2(g) (b) NH4HS(s)  NH3(g) + H2S(g)
(c) Ag (aq) + 2NH3(aq)  [Ag(NH3)2] (aq) (d) HCl(g)  H+(aq) + Cl–(aq)
+ +

(e) 2H+(aq) + Zn(s)  H2(g) + Zn2+(aq) (f) CaF2(s)  Ca2+(aq) + 2F–(aq)


(g) H2O(l) + CO32– (aq) OH–(aq) + HCO3–(aq)

Q.3 Find relationship between KP and KC


(a) 2NH3(g) + 3CuO(s)  3H2O(g) + N2(g) + 3Cu(s)
(b) 2Cu(NO3)2(s)  2CuO(s) + 4NO2(g) + O2(g)
(c) CH4(g) + 2H2S(g)  CS2(g) + 4H2(g)
(d) 3Fe(s) + 4H2O(g)  Fe3O4(s) + 4H2(g)

Q.4 The equilibrium constant at certain reaction at given temperature is K = 103


2NO(g)  N2(g) + O2(g)
Predict the direction of reaction if we start with following partial pressure.
(a) PNO = 0.01 atm, PN 2 = 2 atm , PO 2 = 10 atm

(b) PNO = 10 atm, PN 2 = 0.1 atm , PO 2 = 10 atm

(c) PNO = 20 atm, PN 2 = 50 atm , PO 2 = 8 atm

Q.5 Each of following mixture was placed in a closed container and allowed to stand.
(a) MgCO3(s)
(b) MgO(s) and CO2(g) at a pressure greater than KP
(c) MgCO3(s) and CO2(g) at a pressure greater than KP
(d) MgCO3(s) and MgO(s)
Determine whether or not each mixture can attain the equilibrium : MgCO3(s)  MgO(s) + CO2(g)

Q.6 Given the following information at 298 K :


2NO(g) + Br2(g) 2NOBr(g) KP = 4 atm–1
2NO(g)  N2(g) + O2(g) KP = 4 × 1020
Then Calculate KP for
(a) 4NO (g) + 2Br2(g)  4NOBr(g)
1
(b) NOBr(g)  NO(g) + Br (g)
2 2
(c) N2(g) + O2(g) + Br2(g)  2NOBr(g)
36 Chemistry for JEE

Q.7 A mixture of H2(g) and N2(g) is allowed to attain equilibrium at 427°C. It is found to contain 5 atm H2,
2.5 atm N2 and 0.15 atm NH3. From these data calculate KP and KC for the reaction
N2(g) + 3H2(g)  2NH3(g)

Q.8 A closed system containing 10–2 M H2 and 2 × 10–2 M I2 at 450°C is allowed to reach equilibrium and
at equilibrium the HI concentration is 1.8 × 10–2 M. Calculate KC at 450°C for
H2(g) + I2(g)  2HI(g)

Q.9 Sulfur trioxide decomposes at high temperature in a sealed container :


2SO3(g) 2SO2(g) + O2(g)
Initially, the vessel is charged at 1000 K with SO3(g) at a partial pressure of 0.5 atm.At equilibrium SO3
partial pressure is 0.3 atm. Calculate the value of KP at 1000 K.

Q.10 (a) At 500 K the reaction :


PCl5(g)  PCl3(g) + Cl2(g) KP = 1 atm
In equilibrium mixture at 500 K, the partial pressure of PCl5 is 0.8 atm and that of PCl3 is 0.4 atm. What
is the partial pressure of Cl2 in the equilibrium mixture.
(b) If initially PCl5 is taken at a pressure of 2atm, then what are the equilibrium pressures of PCl3, PCl5
and Cl2 at this temperature?

Q.11 A mixture of 1 mol of NO, 0.5 mol of H2 and 1 mol of H2O is placed in a 1L vessel at 300 K. The
following equilibrium is established.
2NO(g) + 2H2(g)  N2(g) + 2H2O(g)
at equilibrium [NO] = 0.8 M
(a) Calculate KC
(b) Calculate equilibrium concentration of H2,N2 and H2O
(c) Calculate partial pressures of NO, H2, N2 and H2O at equilibrium
[Take : R = 0.08L-atm.mol–1 K–1]

Q.12 A flask is charged with 3 atm of N2O4(g) and 2 atm NO2(g) at 27°C, and following equilibrium is
achieved.
N2O4(g) 2NO2(g)
After equilibrium is reached, the partial pressure NO2 is 1 atm. [R = 0.08 atm-L/mol-K]
(a) What is equilibrium partial pressure of N2O4?
(b) Calculate value of KP for the reaction.
(c) Calculate KC for the reaction.

Q.13 (a)At 1323°C the equilibrium constant for the reaction :


Br2(g) 2Br(g); KC = 10–3
A 0.2Lvessel containing an equilibrium mixture of gases has 0.24 g Br2(g) in it. What is the mass of Br(g)
in the vessel?
(b) For the reaction : H2(g) + I2(g) 2HI(g) KC = 25 at 800 K. In a 2L flask containing an equilibrium
mixture of three gases, there are 0.04 g H2 and 5.08 g I2. What is the mass of HI in the flask?

Q.14 For the following equilibrium at 400K


2NOBr(g)  2NO(g) + Br2(g) KP = 5 atm
If the pressure of NOBr(g) and NO(g) are equal, what is the equilibrium pressure of Br2(g)?
Chemical Equilibrium 37

Q.15 For the following equilibrium at 25°C :


KOH(s) + CO2(g)  KHCO3(s) KP = 6 × 1015 atm–1
if we take 7g KOH and 9g KHCO3 in an evacuated container and allowed to reach equilibrium. Calculate
the pressure of CO2(g) at equilibrium.

Q.16 For the following equilibrium at 400K.


Br2(g) + Cl2(g) 2BrCl(g) ; KC = 9
If 5 mol of Br2 and 5 mol of Cl2 are introduced in 5L container at 400 K. What will be the equilibrium
concentration of Br2, Cl2 and BrCl?

Q.17 Consider the reaction: Cl2(g)  2Cl(g)


A flask is charged with 8 atm of pure Cl2(g), after which it is allowed to reach equilibrium at 400°C.
At equilibrium partial pressure of Cl2(g) is 4 atm.
(a) What is total pressure in the flask at equilibrium
(b) Value of KP for given reaction at 400°C

Q.18 For the following equilibrium at 127°C.


2NOBr(g) 2NO(g) + Br2(g)
An equilibrium mixture in 5Lvessel contains 5.5g NOBr, 3g NO and 8g of Br2
(a) Calculate KC
(b) Calculate total pressure exerted by the vessel.
(c) What was the mass of original sample of NOBr. (If initially only NOBr was present).

Q.19 A sample of solid NH4OCONH2(s) was placed in an evacuated container and then heated so that
following reaction takes place
NH4OCONH2(s)  2NH3(g) + CO2(g)
After heating the total pressure in container was found to be 0.6 atm. Calculate KP for above reaction.

Q.20 For the following reaction at 327°C


NH3(g) + HCl(g)  NH4Cl(s) ; KP = 5 atm–2
(a)An excess of NH4Cl(s) to an empty container and allowed to decompose at 327°C till equilibrium is
reached. Then calculate total pressure at equilibrium.
(b) An excess of NH4Cl(s) is added to a container filled with ammonia at 327°C and 2 atm. Calculate
the pressures of NH3(g) and HCl(g) reached at equilibrium.

Homogeneous equilibria, Degree of dissociation(), Vapour density and Equilibrium constant


Q.21 The vapour density of N2O4 and NO2 mixture at a certain temperature is 30. Calculate the percentage
dissociation of N2O4 at this temperature :
N2O4(g)  2NO2 (g)

Q.22 2 NOBr(g) 2 NO (g) + Br2(g). If nitrosyl bromide (NOBr) is 33.33% dissociated at 25° C & a
total pressure of 0.28 atm. Calculate Kp for the dissociation at this temperature.

Q.23 At 46°C, Kp for the reaction N2O4(g)  2NO2(g) is 0.667 atm. Compute the percent dissociation
of N2O4 at 46° C at a total pressure of 380 torr .

Q.24 The degree of dissociation() of N2O4 into NO2 at 1.5 atmosphere and 40°C is 0.25. Calculate its
Kp at 40°C.Also report degree of dissociation() at 10 atmospheric pressure at same temperature.
38 Chemistry for JEE

Q.25 At some temperature and under a pressure of 4 atm , PCl5 is 10% dissociated. Calculate the
pressure at which PCl5 will be 20% dissociated, temperature remaining same.

Q.26 PCl5 dissociates according to the reaction PCl5  PCl3(g) + Cl2(g) . At 523 K, Kp = 1.78 atm. Find
the density of the equilibrium mixture at a total pressure of 1 atm .

1
Q.27 At 627°C and 1 atm SO3 dissociation as : SO3 (g)  SO2(g) + O (g)
2 2
The density of equilibrium mixture is 0.8 g L–1 . [R = 0.08atm-L-mol–1 K–1]
(a) Calculate KP for the reaction.
(b) Calculate degree of dissociation.

Q.28 When 36.8g N2O4 (g) is introduced into a 1.0-litre flask at 27°C . The following equilibrium
reaction occurs :
N2O4 (g)  2NO2 (g) ; KP = 0.2 atm.
(a) Calculate Kc of the equilibrium reaction.
(b) What are the number of moles of N2O4 and NO2 at equilibrium?
(c) What is the total gas pressure in the flask at equilibrium?
(d) What is the percent dissociation of N2O4?
[Take : R = 0.08L-atm.mol–1 K–1]

Q.29 The system N2O4  2 NO2 maintained in a closed vessel at 60º C & a pressure of 5 atm has an average
(i.e. observed) molecular weight of 69, calculate Kp. At what pressure at the same
temperature would the observed molecular weight be (230/3) ?

Q.30 The degree of dissociation is 0.4 at 400K & 1.0 atm for the gasoeus reaction
PCl5  PCl3 + Cl2(g). Assuming ideal behaviour of all gases. Calculate the density of equilibrium
mixture at 400K & 1.0 atm pressure.

Q.31 In the esterfication C2H5OH(l) + CH3COOH(l)  CH3COOC2H5(l) + H2O(l) an equimolar mixture


of alcohol and acid taken initiallyyields under equilibrium, the water with mole fraction = 0.333. Calculate
the equilibrium constant.

Hetrogeneous Equilibrium
Q.32 Solid Ammonium carbamate dissociates as: NH2 COONH4 (s)  2NH3(g) + CO2(g). In a closed
vessel solid ammonium carbamate is in equilibrium with its dissociation products.At equilibrium, ammonia
is added such that the partial pressure of NH3 at new equilibrium now equals the original total pressure.
Calculate the ratio of total pressure at new equilibrium to that of original total pressure.

Q.33 A sample of CaCO3(s) is introduced into a sealed container of volume 0.821 litre & heated to 1000K
until equilibrium is reached. The equilibrium constant for the reaction CaCO3(s)  CaO(s) + CO2(g) is
4 × 102 atm at this temperature. Calculate the mass of CaO present at equilibrium.

Q.34 Anhydrous calcium chloride is often used as a dessicant. In the presence of excess of CaCl2,, the
amount of the water taken up is governed by Kp = 6.4 × 1085 for the following reaction at room
temperature, CaCl2(s) + 6H2O(g)  CaCl2 .6H2O(s) . What is the equilibrium vapour pressure of
water in a closed vessel that contains CaCl2(s) ?
Chemical Equilibrium 39

Q.35 20.0 grams of CaCO3(s) were placed in a closed vessel, heated & maintained at 727º C under
equilibrium CaCO3(s)  CaO(s) + CO2(g) and it is found that 75 % of CaCO3 was decomposed.
What is the value of Kp ? The volume of the container was 15 litres.

Q.36 At 90°C , the following equilibrium is established :


H2(g) + S(s)  H2S(g) Kp = 6.8 × 102
If 0.2 mol of hydrogen and 1.0 mol of sulphur are heated to 90°C in a 1.0 litre vessel,
what will be the partial pressure of H2S at equilibrium?

Q.37 The equilibrium constant for the reaction is 9.40 at 900°C S2(g) + C(s)  CS2(g) . Calculate the
pressure of two gases at equilibrium, when 1.42 atm of S2 and excess of C(s) come to equilibrium.

Q.38 When 3.06g of solid NH4HS is introduced into a two litre evacuated flask at 27°C, 30% of the solid
decomposes into gaseous ammonia and hydrogen sulphide.
(i) Calculate KC & KP for the reaction at 27°C.
(ii) What would happen to the equilibrium when more solid NH4HS is introduced into the flask?

Le-Chatelier’s Principle
Q.39 Old fashioned “smelling salts” consist of (NH4)2CO3. The reaction for decomposition is :
(NH4)2CO3(s)  2NH3(g) + CO2(g) + H2O(g)
If equilibrium has been achieved, then predict the effect that each of following changes will have on
equilibrium position.
(a) Addition of (NH4)2CO3(s) to the system
(b) Removal of water vapours
(c) If volume of container is doubled
(d) If Ar(g) is added at constant pressure
(e) If tempeature is increased.

Q.40 (a) What would be the effect of an increase in pressure on following equilibria :
(i) CO(g) + 2H2(g)  CH3OH(g) (ii) 2HBr(g)  H2(g) + Br2(g)
(iii) CO2(g) + C(s)  2CO(g) (iv) COCl2(g)  CO(g) + Cl2(g)
(b) In above cases if temperature is increased then what would be the direction of reaction.

Q.41 Following reaction takes place at 127°C and 5L vessel,


SO2Cl2(g)  SO2(g) + Cl2(g) KP = 1 atm
(a) If initially only SO2Cl2(g) was present at 2 atm, then calculate equilibrium partial pressures of each
gas.
(b) If after reaching the equilibrium volume of vessel is doubled then what would be partial pressure of
each gas at new equilibrium.
(c) Calculate vapour density of mixture at new equilibrium.
(d) If inert gas is added at constant volume after attaining new equilibrium till total pressure of mixture
reaches 5 atm then what would be the partial pressure of each gas.

Q.42 For the following decomposition


N2O4(g)  2NO2(g) KP = 8 atm
(a) If equilibrium is established at 16 atm, then calculate partial pressure of NO2 and N2O4 at equilibrium
and also calculate initial pressure of N2O4 if initially only N2O4 was present.
(b) If volume is changed and new equilibrium is established at 50 atm so what would be the partial
pressure of NO2(g) and N2O4(g) at new equilibrium?
40 Chemistry for JEE

Q.43 For the following equilibrium :


AgCl. NH3(s)  AgCl(s) + NH3(g)
(a) What is the effect on partial pressure of NH3 at equilibrium if additional AgCl(s) is added?
(b) What is the effect on partial pressure of NH3 at equilibrium if additional NH3 is pumped into or out
of the system, provided that neither of the two solid phases shown in chemical equation is completely
used up?
(c) What is the effect on partial pressure of NH3 of lowering the tempeature.

Q.44 The Equilibirum constant KC for C(s) + CO2(g)  2CO(g) is 8 at 1000K and 0.25 at 250 K
(a) If excess C is allowed to react with 22 g of CO2 in a 1L vessel at 1000K how many grams of CO are
produced.
(b) How many grams of C are consumed?
(c) Is the reaction endothermic or exothermic?
(d) If a smaller vessel is used for the reaction, will yield of CO be greater or smaller.

Q.45(a) Methanol, a liquid fuel that could possibly replace gasoline, can be prepared from water gas and
additional hydrogen at high temperature and pressure in the presence of a suitable catalyst. Write the
expression for the equilibrium constant for the reversible reaction.
2H2 (g) + CO (g)  CH3OH (g) H = – 90.2 kJ
(b) Assumethat equilibrium has been establishedandpredict how theconcentration of H2, CO and CH3OHwill
differ at a new equilibrium if (1) more H2 is added. (2) CO is removed. (3) CH3OH is added. (4) the pressure
on the system is increased. (5) the temperature of the system is increased. (6) more catalyst is added.

Q.46 Ammonia is a weak base that reacts with water according to the equation
NH3 (aq) + H2O (l)  NH4+(aq) + OH– (aq)
Will any of the following increase the percent of ammonia that is converted to the ammonium ion in
water?
(a) Addition of NaOH. (b)Addition of HCl. (c)Addition of NH4Cl.

Q.47 Suggest two ways in which the equilibrium concentration ofAg+ can be reduced in a solution of Na+,
Cl–, Ag+ and NO3–, in contact with solidAgCl.
Na+ (aq) + Cl– (aq) + Ag+ (aq) + NO3– (aq)  AgCl (s) + Na+ (aq) + NO3– (aq) H = –65.9 kJ

Temperature Dependence of Equilibrium Constant


Q.48 Stearic acid dimerises when dissolved in Hexane
2C17H35COOH(Hexane)  (C17H35COOH)2 (Hexane)
The equilibrium constant for this reaction is 2800 at 27°C but it drops to 40 at 47°C. Calculate H° for
the reaction. (ln 70 = 4.25]

Q.49 For the synthesis of ammonia from its elements


3H2(g) + N2(g)  2NH3(g)
the equilibrium constant K = 6 × 105 at 300 K and H° = – 24 kCal/mol. Calculate equilibrium constant
at 600K, assuming no change in H° between 300 K and 600K. [Given : e–20 = 2 × 10–9]

Q.50 Listed in the table are forward and reverse rate constants for the reaction 2NO (g)  N2(g) +O2(g)
Temperature (K) kf (M–1s–1) kr (M–1s–1)
1400 0.29 1.1 × 10–6
1500 1.3 1.4 × 10–5
Is the reaction endothermic or exothermic? Explain in terms of kinetics.
Chemical Equilibrium 41

Q.51 Rate of disappearance of the reactant A at two different temperature is given byA  B
 d[A]
= (2×10–2 S–1) [A] – 4 × 10–3 S–1[B] ; 300K
dt
 d[A]
= (4×10–2 S–1) [A] –16 × 10–4 [B] ; 400K
dt
Calculate heat of reaction in the given temperature range. When equilibrium is set up.

Miscellaneous
Q.52 For the following equilibrium at 900 K
16
CCl4(g)  C(s) + 2Cl2(g) KP = atm
3
Determine initial pressure of CCl4 that will produce a total pressure of 7 atm.

Q.53 Consider the decomposition of C5H6O3 : C5H6O3(g)  C2H6(g) + 3CO(g)


When a sample of 1 mol pure C5H6O3(g) was taken in an evacuated container of 32L and it is heated to
500 K. The pressure in the flask rise to 2 atm and remained constant at 500 K. Calculate KP for this
reaction.[R = 0.08atm-L-mol–1 K–1]

Q.54 In an evacuated container pure solid NH4HSe is placed at 25°C. Eventually, the pressure above solid
reaches the equilibrium pressure 0.018 atm due to the reaction.
NH4HSe (s)  NH3(g) + H2Se(g)
(a) Calculate KP for this reaction at 25°C
(b) In a different container, the partial pressure of NH3(g) in equilibrium with NH4HSe(s) at 25°C is
0.03 atm. What is the partial pressure of H2Se(g) ?

Q.55 For the following reaction at 100°C


PH3BCl3(s)  PH3(g) + BCl3(g) KC = 10–4 M2
(a) Calculate the equilibrium concentration of PH3 and BCl3 if a solid sample of PH3BCl3 is placed in a
closed vessel at 100°C and decomposes until equilibrium is reached.
(b) If the flask has a volume of 4L, what is the minimum mass of PH3BCl3(s) that must be added to the
flask to achieve equilibrium.

Q.56 A reaction system in equilibrium according to the equation 2 SO2 + O2  2SO3 in 1 litre reaction vessel
at a given temperature was found to contain 0.11 mol of SO2, 0.12 mol of SO3 and 0.05 mol of O2.
Another 1 litre reaction vessel contains 64 g of SO2 at the same temperature. What mass of O2 must be
added to this vessel in order that at equilibrium half of SO2 is oxidised to SO3 ?
Q.57 Amixture of hydrogen & iodine in the mole ratio 1.5 : 1 is maintained at 450ºC.After the attainment of
equilibrium H2(g) + I2(g)  2 HI(g), it is found on analysis that the mole ratio of I2 to HI is 1 : 18.
Calculate the equilibrium constant & the number of moles of each species present under equilibrium, if
initially, 127 grams of iodine were taken.

Q.58 At a certain temperature, CaCO3(s)  CaO (s) + CO2(g) KP = 0.4 atm


C(s) + CO2(g)  2CO(g) KP = 10 atm
Calculate pressure of CO(g) (in atm) at this temperature when excess of C(s), CaCO3(s) and CaO(s)
are mixed together and allowed to attain equilibrium.
Q.59 In a 10 litre container, an equilibrium was established between SO3, SO2 and O2 by starting with only
SO3. The density of equilibrium mixture was found to be 16 gm/litre at a temperature of 900/0.0821K.
If degree of dissociation of SO3 is 40% then calculate the value of equilibrium constant, KP(in atm) for
the reaction. [R = 0.0821 atm-lit./mol kelvin]
2SO3(g) 2SO2(g) + O2(g)
42 Chemistry for JEE

EXERCISE-2 (Objective Questions)


(I) FILL IN THE BLANKS & TRUE OR FALSE:
(i) K for the reaction 2A + B  2C is 1.5 × 1012. This indicates that at equilibrium the concentration of
______ would be maximum.

(ii) The reaction N2 + O2  2NO – Heat, would be favoured by _______ temperature.


1 1
(iii) K for the reaction X2 + Y2  2XY is 100 K. For reaction XY  X2 + Y2 would be _________.
2 2

(iv) Compared to K for the dissociation, 2H2S  2H+ + 2HS–, then K' for the H+ + HS– H2S would have
_______.

(v) The equilibrium constant for a reaction decreases with increase in temperature, the reaction must be
______.

(vi) For the reaction, PCl5(g)  PCl3(g) + Cl2(g), KP and KC are related as ______.

(vii) For the reactions, N2O4(g)  2NO2(g), at equilibrium, increase in pressure shifts the equilibrium in
_______ direction.

(viii) Vant Hoff's equation is _________.

(ix) Dimensions of equilibrium constant, Kc for the reaction 2NH3  N2 + 3H2, are _______.

(x) The value of K for a reaction can be changed by changing _______.

(xi) The degree of dissociation of PCl5 [PCl5(g)  PCl3(g) + Cl2(g)], _________ with increase in pressure
at equilibrium.

(xii) If concentration quotient, Q is greater than KC, the net reaction in taking place in _____ direction.

(xiii) The reaction, N2 + 3H2  2NH3 would be favoured by ____ pressure.

(xiv) Introduction of inert gas at constant volume to a gaseous reaction at equilibrium results in formation of
______ product.

(xv) Van't Hoff's equation gives the quantitative relation between change in value of K with change in
temperature.

(xvi) The value of equilibrium constant changes with change in the initial concentration of the reactants.

(xvii) Extent of a reaction can always be increased by increasing the temperature.

(xviii) For a reaction the value of Q greater than K indicates that the net reaction is proceeding in backward
direction.

(xix) Dissolution of gases in water is high at low temperature.


Chemical Equilibrium 43

[NH3 ]2
(xx) For the reaction, N2 + 3H2  2NH3, the equilibrium expression may be written as K = .
[N 2 ][H 2 ]3

(xxi) For the reaction, CaCO3(s)  CaO(s) + CO2(g), KP = p CO 2 .

(xxii) A catalyst increases the value of the equilibrium constant for a reaction.

(xxiii) If concentration quotient of reaction is less than K, the net reaction is proceeding in the backward
direction.

(xxiv) In case ofendothermic reactions, the equilibrium shifts in backward direction onincreasingthe temperature.

(xxv) The value of K increases with increase in pressure.

(xxvi) For the reaction, H2 + I2  2HI, the equilibrium constant, K is dimensionless.

(xxvii) The reaction 2SO2(g) + O2(g) 2SO3(g), H = –X kJ, is favoured byhigh pressure and high temperature.

(xxviii) A very high value of K indicates that at equilibrium most of the reactants are converted into products.

(II) OBJECTIVE TYPE


Only one is correct
Q.1 The equilibrium constant for the reaction,
N2(g) + O2 (g)  2NO (g) is 5 × 10–4 at 1500 K.
In the presence of a catalyst the equilibrium is attained 8 times faster. Therefore the equilibrium constant
in presence of catalyst at 1500 K is
5
(A) × 10–4 (B) 4 × 10–3 (C) 5 × 10–4 (D) Unpredictable
8

Q.2 Consider the following equilibrium


H2O(g) + CO(g)  H2(g) + CO2(g) K1= 2
FeO(s) + CO(g)  Fe(s) + CO2(g) K2= 4
Then K for reaction.
Fe(s) + H2O(g) FeO(s) + H2(g)
1
(A) 2 (B) 1 (C) (D) 2
2

Q.3 Initiallyfor the equilibrium,


A2(g) + B2(g)  2AB(g)
2 moles of A2 was taken in a 2 litre vessel and 2 mole of B2 was taken in a 3 litre vessel. Both vessel
were then connected.At equilibrium, concentration ofAB(g) is 0.7 M. Equilibrium concentrated ofA2
and B2 gases would be
(A) 0.07 M, 0.07 M (B) 0.05 M, 0.05 M
(C) 0.08 M , 0.08 M (D) 0.06 M, 0.06 M
44 Chemistry for JEE

Q.4 For the following three reactions 1, 2 and 3, equilibrium constants are given :
(1) CO(g) + H2O(g)  CO2(g) + H2(g) ; K1
(2) CH4(g) + H2O(g)  CO(g) + 3H2(g) ; K2
(3) CH4(g) + 2H2O(g)  CO2(g) + 4H2(g) ; K3
Which of the following relations is correct ?
(A) K1 K 2 = K3 (B) K2K3 = K1 (C) K3 = K1K2 (D) K3 · K23K12

Q.5 A definite amount of solid NH4HS is placed in a flask already containing ammonia gas at a certain
temperature and 0.50 atm pressure. NH4HS decomposes to give NH3 and H2S and at equilibrium total
pressure in flask is 0.84 atm. The equilibrium constant for the reaction is :
(A) 0.30 (B) 0.18 (C) 0.17 (D) 0.11

Q.6 Some liquid is taken in an evacuated vessel and the vessel is sealed. Which of the following graph will
correctly represent the achievement of equilibrium of liquid with its vapour at constant temperature?
[re = rate of evaporation, rc = rate of condensation ]

re rc
re rc
r

r
(A) (B) re (C) rc (D) re
rc
t t t t

Q.7 Sulfide ion in alkaline solution reacts with solid sulfur to form polysulfide ions having formulas
S22, S32, S42 and so on. The equilibrium constant for the formation of S22 is 12 ( K1) & for the
formation of S32 is 132 (K2 ), both from S and S2.What is the equilibrium constant for the formation
of S32 from S22 and S?
(A) 11 (B) 12 (C) 132 (D) None of these

Q.8 In a container of 1 L which contains 4 mol of NH3 and 6 moles of H2S at equilibrium. Calculate the
number of moles of H2S that should be added such that at new equilibrium, moles of NH3 decreases by
50% of its value at previous equilibrium.
NH4HS(g)  NH3(g) + H2S(g) KC = 12
(A) 20 (B) 22 (C) 24 (D) 18

Q.9 One mole of N2O4 (g) at 300 K is left in a closed container under one atm . It is heated to 600 K
when 20 % by mass of N2O4 (g) decomposes to NO2 (g) . The resultant pressure is :
(A) 1.2 atm (B) 2.4 atm (C) 2.0 atm (D) 1.0 atm

Q.10 The equilibrium constants K P and K P for the reactions, X  2Y and Z  P + Q, respectively are
1 2

in the ratio of 1 : 9. If the degree of dissociation of X and Z be equal then the ratio of total pressures at
these equilibria is :
(A) 1 : 36 (B) 1 : 1 (C) 1 : 3 (D) 1 : 9

Q.11 When N2O5 is heated at temp. T, it dissociates as N 2 O5  N 2O 3  O 2 , Kc = 2.5. At the same time
N2O3 also decomposes as : N2O3  N2O + O2. If initially 4.0 moles of N2O5 are taken in 1.0 litre flask
and allowed to attain equilibrium, concentration of O2 was formed to be 2.5 M. Equilibrium concentration
of N2O is
(A) 1.0 (B) 1.5 (C) 2.166 (D) 0.334
Chemical Equilibrium 45

Q.12 For the following gases equilibrium :


N2O4 (g)  2NO2 (g)
Kp is found to be equal to Kc. This is attained when temperature is
(A) 0°C (B) 273 K (C) 1 K (D) 12.19 K

Q.13 For the reaction;


2NO2(g)  2NO(g) + O2(g)
Kc = 1.8 × 10–6 at 184° C and R = 0.083 JK–1 mol–1. When Kp and Kc are compared at 184°C, it is
found that :
(A) Kp > Kc (B) Kp < Kc
(C) Kp = Kc (D) Kp  Kc depends upon pressure of gases

Q.14 PCl5 dissociation a closed container as :


PCl5(g)  PCl3(g) + Cl2(g)
If total pressure at equilibrium of the reaction mixture is P and degree of dissociation of PCl5 is , the
partial pressure of PCl3 will be :
    2       
(A) P ·   (B) P ·   (C) P ·   (D) P ·  
   1 1       1 1   

Q.15 For the reaction :


2Hl (g)  H2(g) + I2(g),
the degree of dissociated () of Hl(g) is related to equilibrium constant KP by the expression

1 2 K p 1 2K p 2K p 2 Kp
(A) (B) (C) 1 2K p (D) 1  2 K
2 2 p

Q.16 The equilibrium constant for the reaction


A(g) + 2B(g)  C(g)
is 0.25 dm mol . In a volume of 5 dm3, what amount ofAmust be mixed with 4 mol of B to yield 1 mol
6 –2

of C at equilibrium.
(A) 3 moles (B) 24 moles (C) 26 moles (D) None of these

Q.17 At total equilibrium pressure P1 atm and P2 atm, N2O4 is dissociated to an extent of 33.33% and 50%
P1
respectively. Ratio of P will be
2

3 4 8 3
(A) (B) (C) (D)
8 3 3 4

Q.18 For the reaction in equilibrium


2NOBr(g)  2NO(g) + Br2(g)
if p Br2  P / 9 at equilibrium and P is the total pressure. The ratio KP/P is equal to-
1 1 1 1
(A) (B) (C) (D)
9 81 27 25
46 Chemistry for JEE

Q.19 A 20.0 litre vessel initiallycontains 0.50 mole each of H2 and I2 gases. These substances react and finally
reach an equilibrium condition. Calculate the equilibrium concentration of HI if Keq = 49 for the reaction
H2 + I2  2HI.
(A) 0.78 M (B) 0.039 M (C) 0.033 M (D) 0.021 M

Q.20 A vessel of 250 litre was filled with 0.01 mole of Sb2S3 and 0.01 mole of H2 to attain the equilibrium at
440°C as
Sb2S3 (s) + 3H2 (g)  2Sb (s) + 3H2S (g).
After equilibrium the H2S formed was analysed by dissolving it in water and treating with excess of Pb2+
to give 1.195 g of PbS (Molecular weight = 239) precipitate.
What is value of Kc of the reaction at 440°C?
(A) 1 (B) 2 (C) 4 (D) None of these

Q.21 The equilibrium constant for the reaction CO(g) + H2O(g)  CO2(g) + H2(g) is 3 at 500 K. In a 2 litre
vessel 60 gm of water gas [equimolar mixture of CO(g) and H2(g)] and 90 gm of steam is initially taken.
What is the equilibrium concentration of H2(g) at equilibrium (mole/L)?
(A) 1.75 (B) 3.5 (C) 1.5 (D) 0.75

Q.22 At 87°C, the following equilibrium is established


H2(g) + S(s)  H2S (g) Kp = 7 × 10–2
If 0.50 mole of hydrogen and 1.0 mole of sulfur are heated to 87°C in 1.0 L vessel, what will be the
partial pressure of H2S at equilibrium?
(A) 0.966 atm (B) 1.38 atm (C) 0.0327 atm (D) 9.66 atm

Q.23 For the reaction at equilibrium :


A(g) + 2B(g)  C(g)
Equilibrium constants as function of temperature are
K at 300°C 4 × 10–4
K at 450°C 4.5 × 10–5
K at 600°C 6 × 10–7
Then
(A) Reaction is exothermic.
(B) On adding D(g) at constant volume reaction will move towards right. [D(g) is non reactive gas]
(C) Yield of reaction will increase on increasing temperature.
(D) Both (A) and (C)

Q.24 In which of the following reactions, increase in the pressure at constant temperature does not affect the
moles at equliibrium :
1
(A) 2NH3(g)  N2(g) + 3H2(g) (B) C(g) + O (g)  CO(g)
2 2
1
(C) H2(g) + O (g)  H2O(g) (D) H2(g) + I2(g)  2HI(g)
2 2

Q.25 Which of the following change at equilibrium will shift reaction in backward direction :
Fe3+(aq) + SCN–(aq)  Fe(SCN)2+(aq)
(A)Addition of water
(B)Addition of KOH(aq) [Fe(OH)3 is insoluble in water]
(C) Addition of NaNO3(s)
(D) (A) and (B) both
Chemical Equilibrium 47

Q.26 The conditions favourable for the reaction :


2SO2(g) + O2(g)  2SO3(g) ; H° = – 198 kJ
are :
(A) low temperature, high pressure (B) any value of T and P
(C) low temperature and low pressure (D) high temperature and high pressure

Q.27 For the following equilibrium in a closed rigid vessel.


A (g) B (g) + C (g)
D (g)E (g) + B (g)
If some E (g) is introduced into the vessel, then at the new equilibrium.
(A) [A] increases (B) [C] decreases (C) [A] decreases (D) [B] increases

Q.28 Densities of diamond and graphite are 3.5 and 2.3 gm/mL.
C (diamond)  C (graphite) rH = –1.9 kJ/mole
favourable conditions for formation of diamond are
(A) high pressure and low temperature (B) low pressure and high temperature
(C) high pressure and high temperature (D) low pressure and low temperature

Q.29 The equilibrium SO2Cl2(g)  SO2(g) + Cl2(g) is attained at 25°C in a closed rigid container and an inert
gas, helium is introduced. Which of the following statements is/are correct.
(A) concentrations of SO2, Cl2 and SO2Cl2 do not change
(B) more chlorine is formed
(C) concentration of SO2 is reduced
(D) more SO2Cl2 is formed

Q.30 The yield of product in the reaction


2A(g) + B(g) 2C(g) + Q kJ
would be lower at :
(A) low temperature and low pressure (B) high temperature & high pressure
(C) low temperature and to high pressure (D) high temperature & low pressure

Q.31 Following equilibrium is established at 727°C :


1
SO3(g)  SO2(g) + O (g)
2 2
At equilibrium pressure is 1.2 atm and density of mixture is 0.9 gm/litre.
The degree of dissociation of SO3(g) is - [Given : R = 0.08 atm-litre-mol–1 K–1]
1 2 1 1
(A) (B) (C) (D)
3 3 4 5

Q.32 An exothermic reaction is represented by the graph :

(A) (B) (C) (D)


48 Chemistry for JEE

Q.33 For the reversible reaction :


N2(g) + 3H2(g) 2NH3(g)
at 500°C. The value of Kp is 1.44 × 10–5, when partial pressure is measured in atmospheres. The
corresponding value of Kc with concentration in mol L–1 is :
(A) 1.44 × 10–5 /(0.082 × 500)2 (B) 1.44 × 10–5 /(8.314 × 773)2
(C) 1.44 × 10–5 /(0.082 × 500)2 (D) 1.44 × 10–5 /(0.082 × 773)–2

Q.34 At constant temperature, the equilibrium constant (KP) for the decomposition reaction. N2O4 2NO2
is expressed by KP = 4x2P/(1 – x2) where P is pressure, x is extent of decomposition. Which of the
following statement is true ?
(A) KP increases with increase of P (B) KP increases with increase of x
(C) KP increases with decrease of x (D) KP remains constant with change in P or x

Q.35 Consider the following equilibrium in a closed container :


N2O4(g)  2NO2(g).
At a fixed temperature, the volume of the reaction container is halved. For this change, which of the
following statements holds true regarding the equilibrium constant (KP) and degree of dissociation () :
(A) Neither KP nor  changes (B) Both KP and  change
(C) KP changes, but  does not change (D) KP does not change, but  changes

Q.36 For the reaction :


2NaNO3 (s)  2NaNO2 (s) + O2(g),
the only correct change in equilibrium condition to increase the moles of O2 (g) at new equilibrium is
(A) addition of some NaNO3(s) at initial equilibrium
(B) removal of some NaNO2(s) at initial equilibrium
(C) increase in pressure on system at initial equilibrium
(D) increase in available space over solids at initial equilibrium.

Q.37 Which of the following equilibria is not affected by change in volume of the flask –
(A) PCl5 (g) PCl3(g) + Cl2(g) (B) N2(g) + 3H2 (g) 2NH3(g)
(C) N2 (g) + O2 (g) 2NO (g) (D) SO2Cl2 (g) SO2(g) + Cl2(g)

Q.38 The equilibrium partial pressure of N2O4 (g) and NO2 (g) are 2 atm and 4 atm respectively. If the
volume of container is doubled at fixed temperature then new equilibrium partial pressure of N2O4 (g).
(A) 0.24 atm (B) 1.24 atm (C) 0.76 atm (D) 2.48 atm

Q.39 Which one of the following statements is incorrect about chemical equilibrium-
(A) Chemical equilibrium can be attained, whether we start with reactants or products.
(B) Chemical equilibrium is dynamic in nature.
(C) Chemical equilibrium CaCO3(s)  CaO(s) + CO2(g) is attained when CaCO3(s) is heated in an
open vessel.
(D)At equilibrium concentration of each of the reactants and products become constant.

More than one may be correct


Q.40 For the reaction PCl5(g)  PCl3(g) + Cl2(g), the forward reaction at constant temperature is favoured by
(A) introducing an inert gas at constant volume
(B) introducing chlorine gas at constant volume
(C) introducing an inert gas at constant pressure
(D) introducing PCl5 at constant volume.
Chemical Equilibrium 49

Q.41 For a general substanceAthe phase diagram is represented as shown. Identify the option(s) which is/are
correct :

P 1atm
–3
5×10 atm

200K 400K 500K


T(Kelvin)
(A) Triple point of the substance is (200 K and 3.8 mm of Hg)
(B) Standard boiling point of the substance should be slightly less than 400K.
(C)Above 500K, gas - liquid transition can not occur without change in temperature.
(D) Melting point of the substance will increase with increase in pressure.

Q.42 For followingequilibrium :


2
PNO 2
N2O4(g)  2NO2(g) KP = P
N 2O 4
Graph between Ptotal Vs  is given.

PTotal
(atm)

P1 = 100 atm
P2

0.3 0.4

Where Ptotal = Total pressure at equilibrium
  degree of dissociation ( << 1)
(A) value of KP is 36 atm.
(B) value of P2 is 56.25 atm.
(C) If total pressure is increased at equilibrium then reaction will move in backward direction.
(D) On increasing temperature degree of dissociation of N2O4(g) will increase.

Q.43 Decrease in the pressure for the following equilibria : H2O (s)  H2O(l) result in the :
(A) formation of more H2O (s) (B) formation of more H2O(l)
(C) increase in melting point of H2O(s) (D) decrease in melting point of H2O(s)

Assertion Reason :
Q.44 Statement -1 : Total number ofmoles in aclosedsystem at newequilibriumis less than theoldequilibrium
if some amount of a substance is removed from a system
(consider a reaction A(g) B(g) ) at equilibrium.
Statement -2 : Equilibrium constant of the reaction is changed byremoving some amount of a susbstance
at equilibrium.
(A) Statement-1 is true, statement-2 is true and statement-2 is correct explanation for statement-1.
(B) Statement-1 is true, statement-2 is true and statement-2 is NOT the correct explanation for statement-1.
(C) Statement-1 is false, statement-2 is true.
(D) Statement-1 is true, statement-2 is false.
50 Chemistry for JEE

Q.45 Statement-1 : Ammonia at a pressure of 10 atm and CO2 at a pressure of 20 atm are introduced
into an evacuated chamber. If Kp for the reaction
NH2COONH4 (s)  2NH3 (g) + CO2 (g) is 2020 atm3, the total pressure
after a long time is less than 30 atm.
Statement-2 : Equilibrium can be attained from both directions.
(A) Statement-1 is true, statement-2 is true and statement-2 is correct explanation for statement-1.
(B) Statement-1 is true, statement-2 is true and statement-2 is NOT the correct explanation for statement-1.
(C) Statement-1 is true, statement-2 is false.
(D) Statement-1 is false, statement-2 is true.

Comprehension :
Paragraph for question nos. 46 to 47
If density of a gaseous mixture of dinitrogen tetroxide (N2O4) and nitrogen dioxide (NO2) is 2.5 gm/L at
127° C and 1 atm pressure. [R = 0.08 atm lit/mole-K]
Q.46 Partial pressure of N2O4 is :
(A) 0.62 atm (B) 0.47 atm (C) 0.74 atm (D) 0.26 atm

Q.47 KP for N2O4(g)  2NO2 is :


(A) 0.90 (B) 0.09 (C) 9.0 (D) 0.009

Paragraph for Question no. 48 to 50


Heterogenous equilibria is established in a closed rigid vessel at 1000 K
CS2(g)  C(s) + S2(g) K = 10
Q.48 If initially S2(g) is taken at 11 atm with excess of C(s) then pressure of S2(g) & CS2(g) at equilibrium will
be respectively.
(A) 10 atm, 1 atm (B) 1 atm, 10 atm (C) 5 atm, 0.5 atm (D) 20 atm, 2 atm

Q.49 Reaction will move in forward direction if following change(s) is (are) made at equilibrium
(A) Increase in temperature
(B)Addition of inert gas at constant pressure
(C) Removal of some CS2(g) present at equilibrium
(D) Increase in total pressure

Q.50 Calculate volume of vessel if at equilibrium total moles of CS2(g) and S2(g) are 22.
[Take R = 0.08 atm litre K–1 mol–1]
(A) 40 litre (B) 80 litre (C) 16 litre (D) 160 litre

Match the column :


Q.51 Column I Column II
(A) KP < KC (P) N2 + 3H2  2NH3
(B) Introduction of inert gas at (Q) PCl5 (g)  PCl3 (g) + Cl2 (g)
constant pressure will decrease
the concentration of reactants

(C) K oP is dimensionless (R) 2NO2 (g)  N2O4 (g)

(D) Temperature increase will shift (S) NH3 (g) + HI (g)  NH4I (s)
the reaction on product side.
Chemical Equilibrium 51

Q.52 For the given endothermic reaction


A(g)  2 B(g)
The variation in concentration due to different changes is plotted. In column II various changes are given
and you are supposed to match the entries in column-I with possible changes the system have undergone.
Neglect the slope of change in concentration when system approaches equilibrium.
Assume that changes are carried out very fast.

Concentration

effect-I effect-II effect-III effect-IV

Time
Column I Column II
(The effect is )
(A) Effect-I (P) Addition of inert gas at constant pressure

(B) Effect-II (Q) Decrease in total equilibrium pressure by changing


volume.

(C) Effect-III (R) Decreases in temperature at constant volume

(D) Effect-IV (S) Addition of B only at constant volume

(T) Addition of inert gas at constant volume


52 Chemistry for JEE

EXERCISE-3
SECTION-A
(JEE ADVANCED Previous Year's Questions)

Q.1 If Ag+ + NH3  [Ag(NH3)]+ ; K1 = 1.6 × 103 and [JEE 2006]


[Ag(NH3)]+ + NH3  [Ag(NH3)2]+ ; K2 = 6.8 × 103 .
+
The formation constant of [Ag(NH3)2] is :
(A) 6.08 × 10–6 (B) 6.8 × 10–6 (C) 1.6 × 103 (D) 1.088 × 107

Q.2 The thermal dissociation equilibrium of CaCO3(s) is studied under different conditions.
CaCO3(s) CaO(s) + CO2(g) [JEEAdvance 2013]
For this equilibrium, the correct statement(s) is(are)
(A) H is dependent on T
(B) K is independent of the initial amount of CaCO3
(C) K is dependent on the pressure of CO2 at a given T
(D) H is independent of the catalyst, if any

Q.3 The % yield of ammonia as a function of time in the reaction [JEEAdvance 2015]
N2(g) + 3H2(g) 2NH3 (g) , H < 0
at (P, T1) is given below:

T1
% yield

time
If this reaction is conducted at (P, T2) , with T2 > T1, the % yield of ammonia as a function of time is
represented by :

T2 T1
T1 T2
% yield % yield
(A) (B)

time time

T1 T2
T2 T1
% yield % yield
(C) (D)

time time
Chemical Equilibrium 53

SECTION-B
(JEE Main Previous Year's Questions)
Q.1 What is the equilibrium expression for the reaction P4(S) + 5O2(g) P4O10(s) ?
[AIEEE-2004]
(A) KC = [P4O10]  [P4] [O2]5 (B) KC = [P4O10]  5 [P4] [O2]
(C) KC = [O2]5 (D) KC = 1 [O2]5

Q.2 For the reaction CO(g) + Cl2(g) COCl2(g) the K P is equal to - [AIEEE-2004]
KC

1
(A) (B) RT (C) RT (D) 1.0
RT

Q.3 The equilibrium constant for the reaction N2(g) + O2(g) 2NO(g) at temperature T is
1 1
4×10–4. The value of KC for the reaction NO(g) N2(g) + O at the same temperature
2 2 2(g)
is - [AIEEE-2004]
(A) 2.5 × 102 (B) 50 (C) 4 × 10 –4
(D) 0.02

Q.4 For the reaction : 2 NO2(g) 2 NO(g) + O2(g), (Kc = 1.8 × 10–6 at 184ºC)
When Kp and Kc are compared at 184ºC it is found that (R = 0.0831 kJ/(mol.K))
(A) Kp is less than Kc [AIEEE-2005]
(B) Kp is greater than Kc
(C) Whether Kp is greater than, less than or equal to Kc depends upon the total gas pressure
(D) Kp = Kc

Q.5 The exothermic formaton of ClF3 is represented by the equation :


Cl2(g) + 3F2(g) 2 ClF3(g) ; rH = –329 kJ
Which of the following will increase the quantity of ClF3 in an equilibrium mixture of Cl2, F2 and
ClF3 ? [AIEEE-2005]
(A) Removing Cl2 (B) Increasing the temperature
(C) Adding F2 (D) Increasing the volume of the container

Q.6 A schematic plot of ln Keq versus inverse of temperature for a reaction is shown below
[AIEEE-2005]

The reaction must be


(A) endothermic (B) exothermic
(C) highly spontaneous at ordinary temperature (D) one with negligible enthalpy change

Q.7 An amount of solid NH4HS is placed in a flask already containing ammonia gas at a certain
temperature and 0.50 atm pressure. Ammonium hydrogen sulphide decomposes to yield NH3 and
H2S gases in the flask. When the decomposition reaction reaches equilibrium, the total pressure
in the flask rises to 0.84 atm ? The equilibrium constant for NH4HS decomposition at this temperature
is – [AIEEE-2005]
(A) 0.18 (B) 0.30 (C) 0.11 (D) 0.17
54 Chemistry for JEE

Q.8 Phosphorus pentachloride dissociates as follows, in a closed reaction vessel,


PCl5(g) PCl3(g) + Cl2(g)
If total pressure at equilibrium of the reaction mixture is P and degree of dissociation of PCl5 is
x, the partial pressure of PCl3 will be
[AIEEE 2006]
 2x   x   x   x 
(A)  1 x  P (B)  x  1  P (C)  1 x  P (D)  x  1  P
       

1
Q.9 The equilibrium constant for the reaction SO3(g) SO2(g) + O (g) is Kc = 4.9 × 10–2. The
2 2
value of Kc for the reaction 2SO2 (g) + O2 (g) 2SO3 (g) will be [AIEEE 2006]
(A) 2.40 × 10 –3 (B) 9.8 × 10 –2 (C) 4.9 × 10–2 (D) 416
Q.10 For the following three reactions a, b and c, equilibrium constants are given - [AIEEE 2008]
(a) CO(g) + H2O(g) CO2(g) + H2(g); K1
(b) CH4(g) + H2O(g) CO(g) + 3H2(g); K2
(c) CH4(g) + 2H2O(g) CO2(g) + 4H2(g); K3
Which of the following relations is correct ?
(A) K2 K3 = K1 (B) K3 = K1K2 (C) K3 K23 = K12 (D) K 1 K 2  K 3

Q.11 The equilibrium constants Kp1 and Kp2for the reactions X 2Y and Z P + Q, respectively
are in the ratio of 1 : 9. If the degree of dissociation of X and Z be equal then the ratio of total
pressures at these equilibria is - [AIEEE 2008]
(A) 1 : 1 (B) 1 : 3 (C) 1 : 9 (D) 1 : 36

Q.12 A vessel at 1000 K contains CO2 with a pressure of 0.5 atm. Some of the CO2 is converted into
CO on the addition of graphite. If the total pressure at equilibrium is 0.8 atm, the value of K is:
(A) 1.8 atm (B) 3 atm (C) 0.3 atm (D) 0.18 atm
[JEE Main 2011]

Q.13 The equilibrium constant (KC) for the reaction N2(g) + O2(g)  2NO(g) at temperature T is
4 × 10–4. The value of KC for the reaction, NO(g)  1/2N2(g) + 1/2 O2(g) at the same
temperature is [JEE Main 2012]
(A) 4 × 10 –4 (B) 50.0 (C) 0.02 (D) 2.5 × 102
1
Q.14 For the reaction : SO 2(g)  O 2(g) SO 3(g)' [JEE Main 2014]
2
if KP = KC(RT)x where the symbols have usual meaning then the value of x is (assuming ideality)
1 1
(A) (B) (C) 1 (D) – 1
2 2

Q.15 The standard Gibbs energy change at 300 K for the reaction 2A  B + C is 2494.2 J. At a given time,
1 1
the composition of the reaction mixture is [A] = , [B] = 2 and [C] = . The reaction proceeds in the:
2 2
[R = 8.314 J/K/mol, e = 2.718] [JEE Main 2015]
(A) forward direction because Q < KC (B) reverse direction because Q < KC
(C) forward direction because Q > KC (D) reverse direction because Q > KC
Q.16 The equilibrium constant at 298 K for a reactionA+B  C + D is 100. If the initial concentration of all
the four species were 1 M each, then equilibrium concentration of D (in mol L–1) will be :
(A) 1.182 (B) 0.182 (C) 0.818 (D) 1.818 [JEE Main 2016]
Chemical Equilibrium 55

Q.19 Ethyl acetate is formed bythe reaction between ethanol and acetic acid and the equilibrium is represented
as: CH3COOH (l) + C2H5OH (l)  CH3COOC2H5 (l) + H2O (l)
(i) Write the concentration ratio (reaction quotient), Qc, for this reaction (note: water is not in
excess and is not a solvent in this reaction)
(ii) At 293 K, if one starts with 1.00 mol of acetic acid and 0.18 mol of ethanol, there is 0.171 mol of
ethyl acetate in the final equilibrium mixture. Calculate the equilibrium constant.
(iii) Starting with 0.5 mol of ethanol and 1.0 mol of acetic acid and maintaining it at 293 K, 0.214 mol of
ethyl acetate is found after sometime. Has equilibrium been reached?
Q.20 Describe the effect of :
(a) addition of H2 (b) addition of CH3OH (c) removal of CO
(d) removal of CH3OH on the equilibrium of the reaction: 2H2(g) + CO (g)  CH3OH(g)
Match the column :
Q.21 Match the following equilibria with the corresponding condition
Column I Column II
(i) Liquid Vapour (a) Saturated solution
(ii) Solid Liquid (b) Boilingpoint
(iii) Solid Vapour (c) Sublimation point
(iv) Solute (s) Solute (solution) (d) Melting point
(e) Unsaturated solution
[NH 3 ]2
Q.22 For the reaction : N2(g) + 3H2(g) 2NH3(g), Equilibrium constant Kc =
[N 2 ][H 2 ]3
Some reactions are written below in Column I and their equilibrium constants in terms of Kc are written
in Column II. Match the following reactions with the corresponding equilibrium constnat
Column I (Reaction) Column II (Equilibrium constant)
(i) 2N2(g) + 6H2(g)  4NH3(g) (a) 2Kc
1
(ii) 2NH3(g)  N2(g) + 3H2(g) (b) K c2

1 3 1
(iii) N2(g) + H2(g)  NH3(g) (c) Kc
2 2
(d) Kc2
Q.23 Match the following graphical variation with their description
A B

(i) (a) Variation in product concentration with time

(ii) (b) Reaction at equilibrium

(iii) (c) Variation in reaction concentration with time


56 Chemistry for JEE

EXERCISE-4 (Rank Booster)


[SINGLE CORRECT CHOICE TYPE]
Q.1 Consider following reactions in equilibrium with equilibrium concentration 0.01 M of every species
(I) PCl5 (g)  PCl3(g) + Cl2(g)
(II) 2HI(g)  H2(g) + I2 (g)
(III) N2(g) + 3H2(g)  2NH3(g)
Extent of the reactions taking place is:
(A) I > II > III (B) I < II < III
(C) II < III < I (D) III < I < II

Q.2 For the reaction A(g) + 2B(g)  C(g) + D(g) ; Kc = 1012 .


If the initial moles ofA,B,C and D are 0.5, 1, 0.5 and 3.5 moles respectively in a one litre vessel. What
is the equilibrium concentration of B?
(A) 10–4 (B) 2 × 10–4 (C) 4 ×10–4 (D) 8 × 10–4

Q.3 At certain temperature (T) for the gas phase reaction


2H2O(g) + 2Cl2 (g)  4HCl(g) + O2(g) Kp = 12 × 108 atm
If Cl2, HCl & O2 are mixed in such a manner that the partial pressure of each is 2 atm and the mixture is
brough into contact with excess of liquid water. What would be approximate partial pressure of Cl2
when equilibrium is attained at temperature (T)?
[Given : Vapour pressure of water is 380 mm Hg at temperature (T)]
(A) 3.6 ×10–5 atm (B) 10–4 atm
(C) 3.6 ×10–3 atm (D) 0.01 atm

Q.4 At 675 K, H2(g) and CO2 (g) react to form CO(g) and H2O (g), Kp for the reaction is 0.16. If a mixture
of 0.25 mol of H2(g) and 0.25 mol of CO2 is heated at 675 K, mole % of CO(g) in equilibrium mixture is
:
(A) 7.14 (B) 14.28 (C) 28.57 (D) 33.33

Q.5 Following two equilibrium is simultaneouslyestablished in a container


PCl5(g) PCl3(g) + Cl2(g)
CO(g) + Cl2(g) COCl2(g)
If some Ni(s) is introduced in the container forming Ni (CO)4 (g) then at new equilibrium
(A) PCl3 concentration will increase
(B) PCl3 concentration will decrease
(C) Cl2 concentration will remain same
(D) CO concentration will remain same

Q.6 The equilibrium constant KP (in atm) for the reaction is 9 at 7 atm and 300 K.
A2 (g)  B2(g) + C2 (g)
Calculate the average molar mass (in gm/mol) of an equilibrium mixture.
Given : Molar mass of A2, B2 and C2 are 70, 49 & 21 gm/mol respectively.
(A) 50 (B) 45 (C) 40 (D) 37.5
Chemical Equilibrium 57

Q.7 For the dissociation of N2O5(g) in rigid vessel at constant temperature :


1
N2O5 (g)  2NO2 (g) + O (g)
2 2
If M = Molar mass of N2O5 (g)
D = Vapour density of equilibrium mixture
P° = Initial pressure of N2O5 (g)
Identify the true statement:
2PD
(A) Equilibrium pressure can be expressed as
M

PM
(B) Equilibrium pressure can be expressed as
2D

2(M  D)
(C) Degree of dissociation can be expressed as
3D
(D) Increase in temperature will increase the magnitude of D.

[PARAGRAPH TYPE]
Paragraph for question nos. 8 to 10
Equilibrium constants are given (in atm) for the following reactions at 0° C:
SrCl26H2O(s)  SrCl22H2O (s) + 4H2O(g) Kp = 5 × 1012
Na2HPO412 H2O(s)  Na2HPO47 H2O (s) + 5H2O(g) Kp = 2.43 × 1013
Na2SO410 H2O(s)  Na2SO4(s) + 10 H2O (g) Kp = 1.024 × 1027
The vapor pressure of water at 0°C is 4.56 torr.

Q.8 Which is the most effective drying agent at 0°C?


(A) SrCl2  2H2O (B) Na2HPO47 H2O (C) Na2SO4 (D) all equally

Q.9 At what relative humidities will Na2SO4 10 H2O be efflorescent (release moisture) when exposed to
air at 0°C?
(A) above 33.33% (B) below 33.33 % (C) above 66.66% (D) below 66.66%

Q.10 At what relative humidities will Na2SO4 be deliquescent (i.e. absorb moisture) when exposed to the air
at 0°C?
(A) above 33.33% (B) below 33.33 % (C) above 66.66% (D) below 66.66%

Paragraph for question nos. 11 to 13


In a closed rigid vessel, N2 and H2 gases are taken in 9 : 13 mole ratio, by which the following equilibria
are established :
N2 (g) + 3H2 (g) 2NH3 (g) ; K P1 = ?

N2 (g) + 2H2 (g) N2H4 (g) ; K P2 = ?


At equilibrium, the total pressure is 14 atm, the partial pressure of ammonia is 2 atm and the partial
pressure of hydrogen is 4 atm
58 Chemistry for JEE

Q.11 The value of K P1 is :

1 1 1
(A) atm2 (B) atm 2 (C) 80 atm2 (D) atm 2
80 80 40

1
Q.12 The value of K is :
P2

3 80 40 5
(A) atm 2 (B) atm 2 (C) atm 2 (D) atm 2
80 3 3 3

Q.13 The equilibrium partial pressure of N2 is :


(A) 5 atm (B) 7 atm (C) 3 atm (D) 2 atm

[MULTIPLE CORRECT CHOICE TYPE]


Q.14 When NaNO3 is heated in a closed vessel, oxygen is liberated and NaNO2 is left behind.At equilibrium
(A) addition of NaNO2 favours reverse reaction
(B) addition of NaNO3 favours forward reaction
(C) increasing temperature favours forward reaction
(D) increasing pressure favours reverse reaction

Q.15 Phase diagram of CO2 is shown as following

73
Solid Liquid
67
P (atm)
5

1 Gas

195 217 298 304


T(K)
Based on above find the correct statement(s)
(A) 298K is the normal boiling point of liquid CO2
(B) At 1 atm & 190 K CO2 will exist as gas.
(C) CO2(s) will sublime above 195K under normal atmospheric pressure
(D) Melting point & boiling point of CO2 will increase on increasing pressure

Q.16 The equilibrium between, gaseous isomers A, B and C can be represented as


Reaction Equilibrium constant
A (g)  B (g) : K1 = ?
B (g)  C (g) : K2 = 0.4
C (g)  A (g) : K3 = 0.6
If one mole of Ais taken in a closed vessel of volume 1 litre, then
(A) [A] + [B] + [C] = 1 M at any time of the reactions
(B) Concentration of C is 4.1 M at the attainment equilibrium in all the reactions
1
(C) The value of K1 is
0.24
(D) Isomer [A] is least stable as per thermodynamics.
Chemical Equilibrium 59

Q.17 Consider the equilibrium HgO(s) + 4I– (aq) + H2O (l)  HgI42– (aq) + 2OH– (aq), which changes will
decrease the equilibrium concentration of HgI42–
(A) Addition of 0.1 M HI (aq) (B)Addition of HgO(s)
(C) Addition of H2O (l) (D) Addition of KOH (aq)

[SUBJECTIVE]
Q.18 4.6 gm of liquid ethanol (C2H5OH) is taken in 12 litre container and at 27°C, 40% of ethanol is vaporised
till equilibrium. Now if volume of container is halved and system is allowed to attain equilibrium then find
pressure developed (in atm). [Assume volume of liquid ethanol is neligible]
[Given : R = 0.08 atm lit / mole-K]

Q.19 A mixture of 2 moles of CH4 & 34 gms of H2S was placed in an evacuated container, which was then
heated to & maintained at 727º C. When equilibrium was established in the gaseous reaction
CH4 + 2 H2S  CS2 + 4 H2 the total pressure in the container was 0.92 atm & the partial pressure of
hydrogen was 0.2 atm. What was the volume of the container ?

Q.20 The degree of dissociation of HI at a particular temperature is 0.8 . Find the volume of 1.5M sodium
thiosulphate solution requiredto react completelywith the iodine present at equilibrium in acidic conditions,
when 0.135 mol each of H2 and I2 are heated at 440 K in a closed vessel of capacity 2.0 L.

Q.21 In a closed container nitrogen and hydrogen mixture initiallyin a mole ratio of 1:4 reached equilibrium. It
is found that the half hydrogen is converted to ammonia. If the original pressure was 180 atm, what will
be the partial pressure of ammonia at equilibrium. (There is no change in temperature)

Q.22 At 1200°C, the following equilibrium is established between chlorine atoms & molecule:
Cl2(g)  2Cl (g)
The composition of the equilibrium mixture may be determined bymeasuring the rate of effusion of the
mixture through a pin hole. It is found that at 1200°C and 1 atm pressure the mixtureeffuses 1.16 times
as fast as krypton effuses under the same condition. Calculate the equilibrium constant Kc.

Q.23 SO3 decomposes at a temperature of 1000 K and at a total pressure of 1.642 atm. At equilibrium, the
density of mixture is found to be 1.28 g/l. Find the degree of dissociation of SO3 for
SO3 SO2 + 1/2O2.

Q.24 Consider the equilibrium: P(g) + 2Q(g) R(g). When the reaction is carried out at a certain temperature,
the equilibrium concentration of P and Q are 3M and 4M respectively. When the volume of the vessel is
doubled and the equilibrium is allowed to be reestablished, the concentration of Q is found to be 3M.
Find (A) Kc (B) concentration of R at two equilibrium stages.

Q.25 A saturated solution of iodine in water contains 0.33g I2 / L. More than this can dissolve in a KI solution
because of the following equilibrium : I2(aq) + I aq) I3 aq. A 0.10 M KI solution (0.10 M I)
actually dissolves 12.5 g of iodine/L, most of which is converted to I3.Assuming that the concentration
of I2 in all saturated solutions is the same, calculate the equilibrium constant for the above reaction.
What is the effect of adding water to a clear saturated of I2 in the KI solution ?
60 Chemistry for JEE

Q.26 The equilibrium pXyloquinone + methylene white  pXylohydroquinone + methylene blue may be
studied convinently byobserving the difference in color methylene white and methylene blue. One mmol
of methylene blue was added to 1L of solution that was 0.24 M in pXylohydroquinone and 0.012 M in
pXyloquinone. It was then found that 4% of the added methylene blue was reduced to methylene
white. What is the equilibrium constant of the above reaction? The equation is balanced with one mole
each of 4 substances.

Q.27 A certain gas A polymerizes to a small extent at a given temperature & pressure, nA An . Show that

the gas obeys the approx. equation PV  1  (n  1)K c  where K c  A n  & V is the volume of the
RT  V n1  n
A 
conatiner.Assume that initially one mole ofAwas taken in the container.

Q.28 103 molofCuSO4.5H2Oisintroducedina1.9Lvesselmaintainedataconstanttemperatureof27°Ccontaining


moist air at relative humidity of 12.5%. What is the final molar composition of solid mixture?
ForCuSO4.5H2O(s) CuSO4(s)+5H2O(g),Kp(atm)=10–10.Takevaporpressureofwater at27°Cas28torrs.

Q.29 Two solids X and Y disssociate into gaseous products at a certain temperature as follows:
X(s)  A(g) + C(g), and Y(s)  B(g) + C(g). At a given temperature, pressure over excess
solid X is 40 mm and total pressure over solid Y is 60 mm. Calculate:
(a) the values of Kp for two reactions (in mm)
(b) the ratio of moles ofA and B in the vapour state over a mixture of X and Y.
(c) the total pressure of gases over a mixture of X and Y.

Q.30 When 1 mole ofA(g) is introduced in a closed rigid 1 litre vessel maintained at constant temperature the
following equilibria are established.
A (g)  B(g) + C(g) : KC
1

C (g)  D(g) + B(g) : KC


2

KC [C]eq 1
2
The pressure at equilibrium is twice the initial pressure. Calculate the value of if =
KC [B]eq 5
1

Q.31 Solid NH4I on rapid heating in a closed vessel at 357°C develops a constant pressure of
275 mm Hg owing to partial decomposition of NH4Iinto NH3 and HI but the pressure gradually increases
further (when the excess solid residue remains in the vessel) owing to the dissociation of HI. Calculate
the final pressure developed at equilibrium.
NH4I (s)  NH3(g) + HI(g)
2HI (g)  H2(g) + I2(g), Kc = 0.065 at 357°C

Q.32 2 moles each of A & B were taken in a flask at some temperature so as to establish the following
equilibrium.
A + B  2D
1
it was observed that ofA has reacted. After attainment of equilibria 2 mole of C was added which
3
caused establishment of another equilibria
B+CE+D
1
If at new equilibria mole fraction of E is , then calculate % ofA dissociated after both equilibria has
6
established. Given : 61 7.81
Chemical Equilibrium 61

ANSWER KEY
EXERCISE-1
Q.1 (a) False (b) Ture (c) True (d) True
(e) True (f) True (g) True
PO3 2 [O 2 ]3
Q.2 (a) KP = , KC = (b) KP = PNH3  PH 2S , KC = [NH3] [H2S]
PO2 3 [O3 ]2

[Ag(NH 3 ) 2 ] [H  ][Cl  ] PH 2 [Zn 2 ]


(c) KC = (d) KPC = (e) KPC =
[Ag  ][NH3 ]2 PHCl [H  ]2

[OH  ][HCO 3 ]
(f) KC = [Ca+2] [F–]2 (g) KC =
[CO 32 ]

Q.3 (a) ng = 2 ; KP = KC(RT)2 (b) KP = KC (RT)5 (c) KP = KC (RT)2 (d) KP = KC

Q.4 (a) KP = 103


PN 2  PO 2 210
QP = = = 2 × 105 > KP
PN2 O (0.01) 2
Reaction moves in backward direction.
PN 2  PO 2 0.110
(b) QP = 2
 = 0.01 < KP
PNO 10 2
Reaction moves in forward direction.
PN 2  PO 2 50  8
(c) QP = 2
 = 1 < KP
PNO 20 2
Reaction moves in forward direction.

Q.5 (a) YES (b) YES (c) NO (d) YES


1
Q.6 (a) 16 atm–2 (b) atm–1/2 (c) 10–20 atm–1 Q.7 KP = 7.2 × 10–5 atm–2 ; KC = 0.237 M–2
2
4
Q.8 KC = 29.45 Q.9 KP = atm
90

Q.10 (a) 2 atm (b) PCl3 PCl5 and Cl2 = 1 atm

Q.11 (a) 2.5 M–1 (b) H2 = 0.3M, N2 = 0.1 M H2O = 1.2 M


(c) pNO = 0.8 RT = 19.2 atm; p H 2 = 0.3 RT = 7.2 atm
p N 2 = 0.1 RT = 2.4 atm; p H 2O = 1.2 RT = 1.2 × 0.08 × 300 = 28.8 atm

2 1
Q.12 (a) 3.5 atm (b) atm (c) M Q.13 (a) 0.0432 gm; (b) 12.8 gm
7 84
62 Chemistry for JEE

1 10 15
Q.14 5 atm Q.15 PCO2 = = atm
KP 6
Q.16 Br2 = 0.4 M , Cl2 = 0.4 M, BrCl =1.2M Q.17 (a) 12 atm (b) 16 atm

Q.18 (a) 0.04 M (b) 1.3 atm (c) Mass of NOBr in original sample = 16.5 gm

2
Q.19 3.2 × 10–2 atm3 Q.20 (a) atm (b) PHCl = 0.1, PNH3 = 2.1
5

Q.21 53.33% Q.22 Kp = 0.01 atm Q.23 50%

Q.24 KP = 0.4,  ~ 0.1 Q.25 0.97 atm Q.26 2.7 g / L

Q.27 (a) KP = 1.74 atm1/2 (b) 0.77


Q.28 (a) 8.33 × 10–3 M (b) n NO2 = 0.0556; n N 2O4 = 0.3722 (c) 10.27 atm (d) 6.95 %
Q.29 Kp= 2.5 atm, P = 15 atm Q.30 4.54 g dm–3 Q.31 K=4
Q.32 31/27 Q.33 22.4 mg Q.34 PH 2O = 5 × 1015 atm
Q.35 0.821 atm Q.36 0.379 atm

Q.37 PCS2 = 1.284 atm, PS2 = 0.1365 atm

Q.38 (i) Kc= 8.1 × 10–5 mol2 L2 ; Kp = 4.91 × 10–2 atm2 (ii) Noeffect;

Q.39 (a) No effect (b) Forward reaction (c) Forward reaction


(d) Forward reaction (e) Forward reaction

Q.40 (a) P  V 
(i) Forward (ii) No effect (iii) Backward (iv) Backward
(b) (i) Backward (ii) Forward (iii) Forward (iv) Forward

Q.41 (a) 1 atm of each


(b) PSO 2Cl2 = 0.4 atm; PSO 2  PCl 2 = 0.62 atm
(c) Vapour density = 41.975  42
(d) PSO 2Cl2 = 0.4 atm; PSO 2  PCl 2 = 0.62 atm

Q.42 (a) PNO2 = 8 atm, PN2O4 = 8 atm; Initial PN2O4 = 12 atm

(b) PNO2 = 16.4 atm, PN2O4 = 33.6 atm

Q.43 (a) no effect (b) No change (c) PNH3 will decrease

Q.44 (a) 22.96 gm (b) 4.92 gm (c) Endothermic as on T  K C  es


Chemical Equilibrium 63

1
(d) Q  If V  then Q  ie reaction moves backward and yield of CO decreases.
V

Q.45 (a) K = [CH3OH]/[H2]2[CO] ,


(b) 1. [H2] increase, [CO] decrease, [CH3OH] increase ; 2. [H2] increase, [CO] decrease, [CH3OH]
decrease ; 3. [H2] increase, [CO] increase, [CH3OH] increase ; 4. [H2] increase, [CO] increase,
[CH3OH] increase ; 5. [H2] increase, [CO] increase, [CH3OH] decrease ; 6. no change

Q.46 b

Q.47 Add NaCl or some other salt that produces Cl– in the solution. Cool the solution.

Q.48 – 40.8 KCal Q.49 1.2 × 10–3

Q.50 kr increase more than kf, this means that Ea (reverse) is greater than Ea (forward). The reaction is
exothermic when Ea (reverse) > Ea (forward).

Q.51 16.06 kJ Q.52 5 atm Q.53 10.54 × 10–2 atm3

Q.54 (a) 8.1 × 10–5 atm2 (b) 2.7 × 10–3 atm Q.55 (a) 10–2 M, 10–2 M (b) 6.06 gm

Q.56 9.34 g Q.57 Kc=54, nHI=0.9 mol, nI2= 0.05 mol, nH2 = 0.3 mol

Q.58 2 Q.59 16

EXERCISE-2
1 1
(I) (i) C (ii) high (iii) (iv)
10 K
(v) exothermic (vi) KP = KC (RT) (vii) backward
K2 H  T2  T1 
(viii) log K =   (ix) mol2L–2 (x) temperature
1 2.303R  T2T1 
(xi) decreases (xii) backward (xiii) high
(xiv) same amount of (xv) T (xvi) F (xvii) F
(xviii) T (xix) T (xx) T (xxi) T
(xxii) F (xxiii) F (xxiv) F (xxv) F
(xxvi) T (xxvii) F (xxviii) T

(II)
Q.1 C Q.2 C Q.3 B Q.4 C Q.5 D Q.6 C Q.7 A

Q.8 A Q.9 B Q.10 A Q.11 D Q.12 D Q.13 A Q.14 A

Q.15 D Q.16 C Q.17 C Q.18 B Q.19 B Q.20 A Q.21 A

Q.22 A Q.23 A Q.24 D Q.25 D Q.26 A Q.27 C Q.28 C

Q.29 A Q.30 D Q.31 B Q.32 A Q.33 D Q.34 D Q.35 D


64 Chemistry for JEE

Q.36 D Q.37 C Q.38 C Q.39 C Q.40 CD Q.41 ABC Q.42 ABCD

Q.43 AC Q.44 D Q.45 D Q.46 C Q.47 B Q.48 A Q.49 A

Q.50 D Q.51 (A) P,R,S (B) P,Q,R,S (C) P,Q,R,S (D) Q

Q.52 (A) S (B) T (C) P,Q (D) R


EXERCISE-3
SECTION-A
Q.1 D Q.2 ABD Q.3 B

SECTION-B
Q.1 D Q.2 A Q.3 B Q.4 B Q.5 C Q.6 B Q.7 C

Q.8 D Q.9 D Q.10 B Q.11 D Q.12 A Q.13 B Q.14 A

Q.15 D Q.16 D

You might also like